Этого треда уже нет.
Это копия, сохраненная 5 ноября 2022 года.

Скачать тред: только с превью, с превью и прикрепленными файлами.
Второй вариант может долго скачиваться. Файлы будут только в живых или недавно утонувших тредах. Подробнее

Если вам полезен архив М.Двача, пожертвуйте на оплату сервера.
Тред тупых вопросов №158 697045 В конец треда | Веб
Тред вопросов о жизни, Вселенной и всём таком.

Спрашиваем то, за что в других местах выдают путёвку в биореактор. Здесь анонимные учёные мирового уровня критически рассмотрят любые гениальные идеи и нарисованные в Paint схемы.

Предыдущий тут: >>693926 (OP) (OP)
2 697049
Я так рад что мне выпала волнительная честь задать первый вопросик в новом треде, и тем самым открыть его задав тон характеру треда :3

Масс эффект — это в первую очередь гаремник или все же шутан? И жизнеспособна ли конструкция кораблика НОРМАНДИЯ в реальной жизни?
3 697050
>>697049
Шутан с поебушками.
4 697051
Почему солнце при ближайшем рассмотрении оказывается гречей?
5 697052
>>697051
Среди звезд зарплатных нет.
Не та порода!
6 697053
>>697051
Потому что это газовый шар, в котором постоянно идут какие-то процессы.
7 697055
>>697051
Как гречка совершенная каша, так и наше солнце совершенный объект во вселенной.
8 697056
>>697051
Потому что солнце базовое, и зрелое, много пожило, и много повидало уже. Это не какие-то там порриджевые звезды, это наш, солнечносистемный шар, суровый, в меру горячий, в меру уравновешенный, суровый и не заносчивый.
9 697058
>>697056

>в меру уравновешенный


Но когда постареет его все-таки бомбанет.
10 697059
>>697058
Все так.
11 697061
>>697058
Нихуя, бомбят толстяки с 9 солнечных масс
12 697072
>>697045 (OP)
Фотон — фундаментальная частица, квант электромагнитного излучения в виде поперечных электромагнитных волн и переносчик электромагнитного взаимодействия. Это безмассовая частица, способная существовать только двигаясь со скоростью света.
https://ru.wikipedia.org/wiki/Фотон
E = mc2 (с квадрат).
Подставляем в формулу массу фотона 0 и получаем E каждого фотона = 0 (Ноль).
Если рассматривается свет некоторой определенной частоты ω, то каждый фотон имеет энергию E, пропорциональную этой частоте:
E=ℏω.
Подставляем полученную энергию фотона 0 в формулу и так как постоянная Планка не 0, значит частота каждого фотона 0.

Вопрос: откуда берутся люди(не люди), верующие в это?
13 697074
>>697072
Это другое, понимать надо.
14 697076
>>697058

>Но когда постареет его все-таки бомбанет.


Не бомбанет, Солнце - желтый карлик, желтые карлики не взрываются. Просто аккуратно сбросит внешние слои и останется белый карлик.
15 697083
>>697045 (OP)
Зачем ты вебба прилепил на оп-пикчи, если есть отдельный тред по нему?
16 697099
>>697045 (OP)
Допустим, что E = mc2 верно. Когда электрон испускает фотон, то энергия электрона уменьшается. Согласно этой формуле, также должна уменьшиться и масса электрона.
Электрон (от др.-греч. ἤλεκτρον «янтарь»[5]) — стабильная отрицательно заряженная элементарная частица. Считается фундаментальной (не имеющей, насколько это известно, составных частей) и является одной из основных структурных единиц вещества.
Ну, так что из этого правильное, а что нет?
Какая конкретно фундаментальная частица теряет массу, если она фундаментальная, неделимая и обладает измеренной массой?
И не надо писать всякую бурду, про электронные облака, квантовые пены, протонные каши с кварковыми тефтелями, где "все законы физики совсем другие". Я уже понял, что мочёные в каждом конкретном случает загоняют туфту, противоречащую всей остальной их туфте. Но если тупейшая в истории человечества формула е=mc2 правильна, то где же тупняк про ещё более элементарную частицу, которая аннигилирует и возникает при излучении фотона?
17 697101
>>697099
Наука требует веры. Заткнись и верь.
18 697103
>>697072
У фотона нет массы покоя.
>>697099
Фотон это и есть электрон, но в отличие от бета-частиц (те же электроны) фотоны не образуются в результате радиоактивного распада и не отклоняются в магнитном поле.
19 697104
>>697099
wiki://Предельный случай безмассовой частицы
20 697107
>>697104
>>697104
Что мне толку в ссылках на вики? Я тоже ссылки туда кидаю. В вики написан бред. Вот например по теме масса=энергия
https://ru.wikipedia.org/wiki/Электронвольт
"В физике элементарных частиц в электронвольтах обычно выражается не только энергия Е, но и масса m элементарных частиц[4][5]. Основанием для этого служит тот факт, что в силу эквивалентности массы и энергии выполняется соотношение m = E0/c2, где c — скорость света, E0 — энергия покоящейся частицы".
"Последнее поколение ускорителей элементарных частиц позволяет достичь нескольких триллионов электронвольт (тераэлектронвольт, ТэВ). Один ТэВ приблизительно равен (кинетической) энергии летящего комара[6] или энергии, выделяющейся при падении маленькой капли воды диаметром в 1 мм (массой ок. 0,5 мг) с высоты 3 см".
Значит масса элементарной частицы протон или электрон стаёт как масса комара, который состоит из какого-то количества протонов и электронов. А "масса протона составляет 1,6726⋅10−27 кг или 938,27 МэВ, что примерно в 1836 раз больше массы электрона". И она постоянна.
Полный абсурд. Вера в библию кажется более обоснованной.
21 697108
>>697107
Внимательно прочитай этот раздел из статьи про эквивалентность массы-энергии. Следующий тоже не помешает.
Если не поймёшь, ничего страшного. Иди читать историю науки, про ультрафиолетовую катастрофу и как её решали. И что из этого получилось, и на какие грабли наступали.
Если там будет скучно, сходи, ну я не знаю, на лурку. Научпоп так себе, на уровне спичек и желудей, но хоть что-то.
Ещё вариант — сходить нахуй, только учти, что и у хуя есть своя собственная энергия покоя.
22 697109
>>697107
Масса есть, иначе не было бы эффекта гравитационного линзирования звезд, просто она маленькая и ей пренебрегают при других расчетах. Это как не учитывают массу воздуха в пакетике с чипсами, хотя поток того же воздуха вон ветрогенераторы двигает.
23 697110
>>697108
Если запись, что протон в ускорителе увеличивает свою массу на 24 порядка, оставаясь при этом элементарной частицей с постоянной массой, кажется логичной какому-то человеку, он сразу должен получать в подарок литр галоперидола внутривенно.
Читать теорию по физике элементарных частиц, астрофизике = сходить нахуй.
24 697113
>>697110
А ты ученый, чтобы сомневаться?
25 697120
>>697103

>Фотон это и есть электрон


>в отличие от бета-частиц (те же электроны) фотоны не


Дегенерат, спок.
28265.jpg610 Кб, 900x674
26 697122
>>697083
Заткнись, лицезрей совершенство
27 697124
Квантовые флуктуации откуда взялись?
28 697125
>>697124
Хз. Нужно дальше считать и улучшать показометры.
16394193134330.png4 Кб, 576x448
29 697126
>>697099

> Я уже понял, что мочёные в каждом конкретном случает загоняют туфту,


ВСЁ СОСТОИТ ИЗ МОЛЕКУЛ
@
МОЛЕКУЛЫ СОСТОЯТ ИЗ АТОМОВ
@
АТОМЫ СОСТОЯТ ИЗ ПРОТОНОВ И НЕЙТРОНОВ
@
ПРОТОНЫ И НЕЙТРОНЫ СОСТОЯТ ИЗ КВАРКОВ
@
КВАРКИ СОСТОЯТ ИЗ ХУЙНЯНЕЙМ
@
ХУЙНЯНЕЙМ СОСТОИТ ИЗ КВАНТОВОЙ ХУЙНЯНЕЙМ И ТАК ДО БЕСКОНЕЧНОСТИ
@
ДАЙТЕ ДЕНЕХ
image.png1,5 Мб, 1200x900
30 697140
Вы уже видели новую фотографию солнца с зонда Паркер?
15955137891-p-prometei-film-1.jpg289 Кб, 1468x972
31 697149
Поясните нахуй лететь к Марсу если есть Титан? Почему бы Роскосмосу когда допилит буксир не полететь туда? Скорости же должно хватить, можно сбросить зонды и они будут далеко не маленького размера, буксир же может дохуя тащить. А потом уебать буксир в межзвездное пространство как вояджеры
32 697151
>>697140
Камешки какие-то
33 697153
>>697045 (OP)
>>697122
Какая вероятность обнаружить пиздецому Солнечной системе в виде потухшей нейтронной звезды с этого "совершенства" ?
34 697154
>>697110
Ладно, покормлю, ТТВ или не ТТВ. Корень твоих заблуждений в неразличении инвариантной и релятивистской массы.
Хотя в этом контексте говорить о массе вместо более правильного отношения 4-импульса к 4-скорости не очень полезно. Это упрощение только приводит к заблуждениям и даже для научпопа не полезно.

https://ru.wikipedia.org/wiki/Масса_в_специальной_теории_относительности
35 697155
>>697153
Нейтронная звезда остывает очень медленно: для полного исчерпания энергии требуется 10 в 16 степени или даже 10 в 22 степени лет, то бишь ни как, следую теории ТБВ
36 697156
>>697155
Если остывание такое медленное, почему говорят про эволюцию нейтронных звёзд? Насколько малое нейтронное тело можно получить перекачкой материи двойной звезды с её нейтронной части и находили ли что-то похдобное?
37 697157
>>697140
Греча золотая. Зохавал бы.
38 697159
>>697149
Марс ближе чем титан. Мы были на титане, и будем еще (как минимум драгонфлай, или тссм можно возобновить).

>Почему бы Роскосмосу


Не пойти нахуй. Пусть хотя бы глоб запустят и не обосрутся. Им некогда наукой заниматься, еще лет 10 после глоба будут хуи пинать.
39 697161
>>697045 (OP)
В РИТЭГах используют зачастую Плутоний-238 который распадается в свинец выделяя энергию один раз.
А нет ли ритэгов где многоступенчатый распад чтобы продукт распада тоже потом выделял энергию и еще и еще?
Какой-нибудь Кюрий, Америций, Беркелий, что-нибудь такое не рассматривали? Я не разбираюсь, извините за тупость.
40 697163
>>697161
Многоступенчатый распад нет.
Америций-241 рассматривали, ибо у него плотность энергии охуенная. Его вообще для чего только не рассматривали. Но толку-то, если его получить можно как с козла молока.
Ещё рассматривали ядерно-оптические преобразователи. Распад частиц возбуждает газ, возбуждённый газ светит в фотовольтаическую панель, которая имеет охуевшую эффективность из-за заточенности под одну узкую полоску спектра. Из плюсов - плотность энергии как у бензина (т.е. во много раз больше ритэга), и решение проблем с термодинамикой (эта хуйня не так сильно греется). Из минусов - 100 атмосфер радиоактивного пердежа в баллоне, при бабахе превращающие всю эту затею в грязную бомбу.

>>697159
РК вообще не занимается наукой. Наукой занимается соответствующая научная организация. И это разделение - нихуёвый такой стопор, особенно с таким отношением к науке.
41 697164
>>697163

>242


фикс, конечно
42 697167
>>697140
Пересветили фоточку-то. Не рассчитали светопоток заранее, что ли.
Алсо, гораздо более впечатляющая вот эта хуйня. https://www.youtube.com/watch?v=IQXNqhQzBLM
43 697169
>>697167
Это рофл, на картинке буквально отфотошопленная гречка.
44 697170
>>697169
Лол. А фотки с пролёта-то есть вообще? Если таймлапс передали, то и прямые фотки поверхности должны были.
45 697171
>>697163
Грубо говоря, ты прав. Они не занимаются ноукой по определению. Под "занятием ноукой" я подразумеваю прожекты, у которых хоть какие-то околонаучные цели есть. Вроде глоба, венеры-д, и прочих маняфантазий. Ну глоб еще куда ни шло, а вот венеры нам не видать еще лет 20 видимо. Ебаные политиканы.
46 697180
>>697120
Ты хули на отца тявкаешь, щенок. Домой придешь пороть буду.
47 697182
>>697153
Нет никакой нейтронки в Солнечной Системе.
image.png207 Кб, 549x384
sage 48 697184
49 697187
>>697099

>Согласно этой формуле, также должна уменьшиться и масса электрона.


Эквивалентность массы и энергии не означает, что это одна и та же сущность. Когда у тебя телефон разряжен, энергии в нем меньше, чем когда он заряжен, однако масса у него в обоих случаях одинакова и не меняется при изменении энергии. Короче, очень тухлый проход в демагогию. Иди на хуях кататься, вероблядь.
50 697188
>>697124
Из ниоткуда.
51 697189
>>697103

>Фотон это и есть электрон


А электрон есть хуен. Бананами иди жонглировать, кловен ёбаный.
52 697190
>>697153
Нулевая, потому что в Солнечной системе никаких нейтронных звезд нет.
53 697193
>>697184
Ну ты представляешь, какими были бы орбиты планет в нашей системе, если бы где-то рядом пролетел объект с ~солнечной массой?
54 697196
>>697187

>однако масса у него в обоих случаях одинакова и не меняется при изменении энергии


Пиздабол, спок.
55 697197
>>697109
"Масса есть"
Неуч, нахуя ты пишешь в этой теме?
"Все частицы,масса которых равна нулю,двигаются со скоростью света и,наоборот,все частицы,которые двигаются со скоростью света,имеют массу равную нулю.Так что ответ такой: это свойство частиц,двигающихся со скоростью света. Физический смысл такого утверждения состоит в том,что массой частицы наделяет поле Хиггса и,соответственно,тормозятся им до скоростей меньших скорости света.Ну,а безмассовые частицы потому и безмассовые,что с полем Хиггса не взаимодействуют,поэтому и двигаются со скоростью света".
56 697199
>>697154
Ты мало пишешь, поэтому беру псевдонаучный высер:
https://yandex.ru/q/question/pochemu_fotony_ne_imeet_massy_e702dd13/
Если масса покоя и масса релятивиста (шизика) - это разные физические величины, которые абсолютно независимы, то какого хрена в формуле е=мц2 написана масса? Если бы имели ввиду что-то другое, то назвали бы другим словом, например, с ростом скорости растет импульс. Но нет же, множат граммы на скорость света в квадрате.
С какого хрена эта формула справедлива только для покоящихся тел? Есть система отсчёта, где фотон покоится, а всё вокруг него движется. ПОКОЯЩИХСЯ ТЕЛ НЕ СУЩЕСТВУЕТ. Если формула неприменима к фотону, значит она неприменима НИ К ЧЕМУ.
57 697200
h
image.png327 Кб, 680x680
58 697201
Почему не вырабатывают энергию ядерными отходами?
59 697202
>>697109

>Масса есть, иначе не было бы эффекта гравитационного линзирования звезд


Ты что, в церковной школе закончил 8 классов? Если бы у фотона была хоть какая-то масса, то свет от звезд, летящий миллиарды лет, отклонялся бы всеми космическими телами, при чём на разные фотоны действие гравитации было бы разное, из-за разного расстояния. А тогда бы размывались границы звёзд, и НАСА бы постеснялось рисовать свои картиночки. (Думаю, они не постеснялись рисовать картиночки, зная что там что-то другое. Например, бесконечная тьма и пустота. Очевидно, что наш мир имеет границы. Почему бы им не быть совсем близко?).
sage 60 697204
>>697193

>Ну ты представляешь


>где-то рядом пролетел


Задача о трёх телах не имеет решения, представить такое трудно. Гравитационные возмущения орбит планет на протяжении лярдов лет "успокаивает" тело большой массы, которое должно имееть эксцентриситиет около единицы; оно же изменило орбиты плутоноидов и причина наклона солнечной оси вращения. Пояс астероидов и массовые вимирания, возможно, тоже из-за сближения к Солнцу этим телом. Период орбиты такого тела десятки тысяч лет офк. Перифокусное расстояние, если это нейтронная звезда, может быть довольно большим кстати.
>>697190
Чайник Рассела, плес.
61 697205
>>697049
Baнильное кинцо для казуалов
62 697214
>>696843 →
>>696813 →
Я в курсе этих басней про каналы, только один вопрос: как это относится к академической науке? Ты ещё уфологию в науку запиши.
63 697215
ИЗВЛЕЧЕНИЕ ИНФОРМАЦИИ ИЗ ЧЕРНОЙ ДЫРЫ
Как нам всем говорят, ничто не может покинуть черную дыру. Но почти все забывают тот факт, что гравитация-то блять её покидает.

Значит, возможно, можно передавать информацию из-под горизонта с помощью тех же гравитационных волн.
Не имеет значения имеет , что есть на самом деле гравитация, искривление пятимерного пространства, или же само пространство-время является проявлением каких-то квантовых эффектов.
Вопрос в том, насколько ровное гравитационное поле у ЧД.

Если процессами внутри дыры можно повлиять на ее внешнюю гравитацию, то и информацию передавать можно.

Для этого нужны экспериментальные данные градиента величины гравитационного поля вокруг ЧД с крайне высокой точностью.

Ну или, хотя бы, надо бросать всякое тяжёлое говно (вроде планеты) в ЧД и смотреть на изменение гравитации вокруг нее. Если поле будет изменяться неравномерно, то это 100%-й пруф возможности извлечения информации.
64 697219
>>697202

>Если бы у фотона была хоть какая-то масса, то свет от звезд, летящий миллиарды лет, отклонялся бы всеми космическими телами, при чём на разные фотоны действие гравитации было бы разное, из-за разного расстояния.


Какая удивительная вселенная, вот бы там побывать.
>>697204

>Гравитационные возмущения орбит планет


Это которые возникли при прохождении сквозь сс гипотетического массивного объекта или какие?

>эксцентриситиет около единицы


С 1.0 я бы согласился. Звезда пролетела, столкнула Нептун, съебалась.

>Пояс астероидов


Который именно?

>массовые вимирания


Периодичность мнимая.

>Период орбиты такого тела десятки тысяч лет


Маловато было бы, имо были бы более выраженные аномалии.
Вообще-то теория неплохая, показала бы бесполезность концепции тёмной материи.
1640021987690.jpg22 Кб, 464x401
65 697220
>>697202

> свет от звезд, летящий миллиарды лет, отклонялся бы всеми космическими телами


> при чём на разные фотоны действие гравитации было бы разное, из-за разного расстояния


Так и есть
66 697237
1) Анчоусы, а можно ли что-то видеть внутри чёрной дыры? Вот лечу я такой в ЧД, пересёк горизонт событий. Я смогу увидеть что-то (всё что туда упало до меня?), прежде чем попаду в сингулярность и стану героем?

2) Сторонний наблюдатель, как я понял, никогда не увидит как объект погружается под горизонт событий. Т.Е., он будет двигаться всё медленнее и медленнее пока окончательно не замрёт. Но ведь тогда сторонний наблюдать должен видеть ВСЕ объекты, которые когда либо упали в ЧД, размазанными по горизонту событий! Тогда получается, что ЧД должны быть наблюдаемый в телескоп ибо на их поверхность должны быть намотаны все объекты, когда-либо упавшие туда.

3) Человек, падающий в ЧД при подлёте к горизонту событий может увидеть всё будущее Вселенной, если оглянётся назад, так? Ну, теоретически. Но если он увидит всё будущее Вселенной, то он увидит и как ЧД испарилась под действием излучения Хокинга. Тогда выходит что чёрной дыры рядом с ним уже не будет, т.е. он никогда не уйдёт под горизонт событий?
67 697241
>>697237
https://www.youtube.com/watch?v=S6qw5_YA8iE

Таскай изображение мышкой, чтобы менять угол зрения если нищеёб без VR-шлема Хуйню про увидеть будущее забудь.
image.png958 Кб, 1144x1232
68 697242
>>697241
Почему оно не круглое?
Алсо как перестать бояться?
69 697244
>>697220
Он не "отклоняется". Фотоны всегда, ВСЕГДА летят по кратчайшему пути мировой линии. Это пространство такое досталось всратое, в смысле не плоское, а погнутое местами. Б/у, короче.
70 697245
>>697242

>Почему оно не круглое?


Из-за вращения дыры.

>Алсо как перестать бояться?


Перестать вскрывать темы, которые не стоит вскрывать. Вы молодые, шутливые, вам все легко. Это не то.
71 697249
>>697244
>>697244
пространство и время всегда нормальные, есть притяжение (хуй проссы как оно работает, но никаких гравитационных волн нету нахуй), а всякие остановка времени - это просто замедление физ и хим процессов, а увидеть будущее или улететь в прошлое - это ваще пиздец. Будущее ещё не случилось, а в прошлое - типа вся вселенная должна остановиться и начать ехать взад штоли? да с хуяли она поедет?
72 697253
>>697201
Ты вообще не туда этот вопрос задаешь. Но пусть так. Потому что это вызовет еще больше проблем, технического, экологического и многих других планов. Если ты с ядерной физикой на "ваше величество", то посмотри ролик бояршинова на ютабе, он там подробно расписал про "сосать и перспективки". Причем весьма доступно.
>>697214
К науке? Никак и как одновременно. Четверть доски инопланетян ищет, кометы космическими кораблями воображает, и в плоскую землю верит. А потом приходит коммишизик и говорит, что это все из-за капитализма. Приходится и с этими разговаривать.
В чём он не прав? 73 697254
В двух словах о твоем «материализме» и состоянии современной науки. В действительности ты мыслишь в рамках механистической физической картины мира образца 19 века. Но в 20 веке появилась квантовая физика, согласно которой (в интерпретации Бома) наша предметная действительность, явный или развернутый порядок - это просто иллюзия, а реальность представляет собой голографическое свертывание и развертывание, которое возникает из более глубокого скрытого порядка. А современная нейробиология, опирающаяся на идеи Бома и голографический принцип, говорит нам что мы живем в искусственно сконструированной нашим мозгом реальности, который в свою очередь обрабатывает частоты, пришедшие из более глубокого порядка существования, находящегося за пределами пространства и времени. Т.е. и разум, и материя находятся, в конечном итоге, в скрытых порядках, и что во всех случаях явные порядки проявляются как относительно автономные, раздельные и независимые объекты, сущность и формы, развёртывающиеся из скрытых порядков. Ну и конечно, исходя из этого совершенно закономерно что многие создатели квантовой физики, такие как Шрёдингер и Гейзенберг, были метафизиками в той же степени, в которой были физиками, и писали про Абсолют и про Брахман в своих трудах
74 697256
>>697254
Репорт антисетипетуха.
75 697258
>>697256
Это твой протык? Паста с религача тащемта
76 697264
>>697056
Да. Только голосует за большой атрактор, когда его туда свозят на гравитационных автобусах
77 697270
>>697249

> пространство и время всегда нормальные, есть притяжение (хуй проссы как оно работает, но никаких гравитационных волн нету нахуй)



> есть притяжение



> хуй проссы как оно работает



Тупых ответов тред, 1904 года разлива.
78 697272
>>697196
Вероблядь, не гори.
79 697274
>>697215

>гравитация-то блять её покидает.


Не покидает, ты обосрался.
80 697275
>>697274
Вероблядь, тушись.
81 697276
>>697237

>1) Анчоусы, а можно ли что-то видеть внутри чёрной дыры?


Да, фотоны из внешней Вселенной благополучно будут попадать тебе на глаза через горизонт. Именно поэтому ты даже не заметишь, что оказался за горизонтом, т.к. это не физическая поверхность.

>Но ведь тогда сторонний наблюдать должен видеть ВСЕ объекты, которые когда либо упали в ЧД


Нет, не должен, потому что из-за эффекта гравитационного красного смещения фотоны от объекта на горизонте приходят покрасневшими. Там буквально за доли секунды сигнал станет настолько слабым, что его нельзя будет никак зафиксировать. Любой объект на горизонте для стороннего наблюдателя сначала покраснеет, а потом исчезнет.

>3) Человек, падающий в ЧД при подлёте к горизонту событий может увидеть всё будущее Вселенной, если оглянётся назад, так?


Нет.
82 697277
>>697254
Шиз, таблетки.
83 697278
>>697275
Вероблядь, спок.
84 697282
>>697215
Для начала, уничтожение информации в ЧД это всё ещё не решённый вопрос.

>Вопрос в том, насколько ровное гравитационное поле у ЧД.


Эксперимент, нужный для ответа на это, малореально захуячить на практике. При расстояниях до реальных ЧД, на доступных даже Лизе базах понадобится целая сеть детекторов, на дохуя порядков более чувствительных чем есть.
Вот если бы подобраться хотя бы на несколько сот а.е, было бы как нехуй измерить.
sage 85 697290
>>697272
>>697275
>>697277
>>697278
Зарепортил выблядка.
86 697324
>>697045 (OP)

>wiredplaceholde[...].jpg


А нельзя выдувать зеркала прямо на орбите, типо как стеклодувы выдувают стеклянные шары? Солнцем плавить стекло, выдувать будет сам вакуум, потом просто покрыть одну сторону пузыря серебристым налетом и установить камеру в фокусной точке?
87 697328
У всех ли звезд есть планеты? Вот например смотришь ночью на небо и там дохуя звезд. И что у каждой той звезды тоже есть планеты крутящиеся вокруг неё, мб даже потенциальной с жизнью на них?
88 697331
>>697328
У некоторых может и не быть. Раньше вообще не думали что планет так дохуя, но вот наоткрывали тысячи уже, так что звездообразование зачастую подразумевает еще и протопланетный диск из которого каменюки и газонюхи получаются. Вот катаклизменное звездообразование или какое событие типа мимопролетала другой звезды может нарушить это и планет не окажется.
89 697334
>>697324
Вопрос типа: нельзя ли вместо тюбиков с фаршем, разводить на орбите коров? Ответ: да, можно. Но дешевле и проще все делать на земле. Пока что.
90 697339
>>697334
Идиотская аналогия. В случае с зеркалом мы доставляем количественно ту же массу стекла, но выигрываем в объеме.
91 697341
>>697324
Уже было >>694135 →
Чуть лучше, но смысл тот же самый. Если не обеспечить точность в доли длины волны, то разрешающая способность будет никакая.
92 697345
>>697339

> Идиотская аналогия.


Каков вопрос в треде тупых вопросов, такой и ответ
93 697350
>>697339
Нет, аналогия хорошая, это ты идиот.
94 697356
>>697339
А также стекольный завод.
95 697359
>>697328

>У всех ли звезд есть планеты?


Скорее нет, чем да (Гипотетическая Немезида, как гипотетический пример такой гипотетической звезды)

>Вот например смотришь ночью на небо и там дохуя звезд


Так.

>И что у каждой той звезды тоже есть планеты крутящиеся вокруг неё


Вовсе не обязательно.

>мб даже потенциальной с жизнью на них?


Совсем не обязательно. Солнцеподобные звезды это одно. Красные карлики это совсем другое. Еще зона обитаемости и куча других факторов, для того чтобы хотя бы гипотетически предполагать жизнь вокруг звезды.
96 697360
возмём некую солнечную систему, где есть планета массой и размером с юпитер, но катающаяся на орбите между земной и марсинской. у этого юпитера 3 спутника и все три - марсы. марсы вращаются близко друг к другу и делают рокировки как янус и япет, из-за чего каждый из трёх марсов успевает побывать и на ближней, и на средней и на дальней орбитах. теперь, указав исходные данные, задаю тупые вопросы:
- марсы эти явно должны быть достаточно близко к друг другу для рокировки орбит. насколько?
- марсы приливно захвачены, как и всякий спутник. будут ли они поворачиваться и проворачиваться при рокировке орбит?
- можно ли рассчитать время нахождения конкретно взятого марса на данной орбите и рассчитать полный цикл скакания по орбитам и возвращения на исходную точку положения? если брать рассчёт не временем, а оборотами, например
- инициируют ли рокировки марсов магнитное поле? постоянное или хотя бы возникающее при рокировке что в условиях относительно маленькой орбиты у юпитера будет случаться раз в считанные часы
97 697367
>>697360
у джуповского протыка ганимеда и без всяких рокировок есть поле. а у ио почему-то нету, хотя ио это голое ядро ганимеда и железные ядра у них +/- одинаковые. есть версия что из за магма-оушена ио перегрелась и ядро не остывает (т.к. магма оушен такой же горячий как ядро, поэтому в ядре нет конвекции и магнитного поля нет. а у ганимеда всё как положено. короче, если у марсов будут магма-оушены то тютю
98 697375
>>697328
По идее у большинства звезд должны быть. В протопланетном диске большая часть материи уходит на звезду, но статистически всегда остается мелкий мусор, из которого что-нибудь да слипнется, не планеты, так хоть астероиды.
Исключение тут - это молодые новорожденные звезды, у которых планет еще не образовалось.
А вот с жизнеспособностью - ну как повезет, скорее всего это редкость.
99 697382
>>697045 (OP)
Если Луна всего лишь отражает свет Солнца, то почему она ярче, чем песок в моём дворе?
Очень хочу узнать очередную шизо теорию про какой-нибудь эффект лунно-фотонного попердоливания пространство-говна, который как-будто бы объясняет это.
100 697386
>>697045 (OP)
https://birdinflight.com/ru/mir/hvatit-vrat-novye-foto-vysadki-na-lune.html
Статья "доказывает", что всё про Луну - правда."Солнце нагревает поверхность Луны до 120°C, но используемая астронавтами плёнка на специальной термостойкой основе могла начать плавиться только при 260°C".
А почему вакуумный солнечный коллектор на Земле очень легко нагревается выше 200° и срабатывает клапан сброса давления, а если нет, то нагревается, пока не лопнет, а вот на Луне, где атмосфера не поглощает излучение Солнца, жалкие 120 градусов?
101 697387
В удачный день тёмный камешек нагреется до 120 и в Египте. Получается, что у Луны все эти атмосферы, магнитосферы и прочие сферы идентичны таковым на Земле? Мы же не можем сомневаться, что Армстронг ходил в резиновых галошах при 120 градусах, ведь это подтвердили "всестороннюю документацию высадки, в том числе специалистами конкурирующего с НАСА космического ведомства СССР — данные радаров, наблюдения через телескопы, принятые сигналы с борта, записи разговоры астронавтов, телевизионную картинку". Не может же это всё быть враньём.
102 697389
>>697382
Возможно дело просто в большой площади?
103 697390
/test
104 697391
>>697382
>>697386

>заговорчики в /zog/

105 697393
>>697387
А фокусирующая линза на Земле вообще до температуры нагрева бумаги нагреет. Мысль ясна?
106 697395
>>697328

>У всех ли звезд есть планеты?


Почти у всех.
107 697396
>>697382

>Если Луна всего лишь отражает свет Солнца, то почему она ярче, чем песок в моём дворе?


Твой песок ночью нихуя не отражает, кроме слабого переотраженного Луной солнечного света. Ему не от чего быть ярким.
108 697400
>>697387

>Мы же не можем сомневаться, что Армстронг ходил в резиновых галошах при 120 градусах


Про резиновые галоши фабрики Треугольник тебе сообщил лично Армстронг?
109 697401
>>697393

>А фокусирующая линза на Земле вообще до температуры нагрева бумаги нагреет. Мысль ясна?


Ясно, что у тебя умственная отсталость тяжёлой степени.
110 697402
>>697367

>если у марсов будут магма-оушены то тютю


Океан магмы есть и у Земли. А знаешь, что ещё есть у Земли? Поле.
111 697423
>>697382

>почему она ярче, чем песок в моём дворе?


Чем замерял, когда и сколько раз замеры делал? Покажи выкладки.
Бьюсь об заклад - ошибка в измерениях.
112 697424
>>697386
Солнечный коллектор может нагреваться хоть до пяти тысяч градусов, потому что он коллектор. Без коллектора на сраной земляшке нагрева выше 80 градусов цельсия не наблюдается.
image.png131 Кб, 3081x1441
113 697425
>>697387

>Мы же не можем сомневаться, что Армстронг ходил в резиновых галошах при 120 градусах, ведь это подтвердили "всестороннюю документацию высадки, в том числе специалистами конкурирующего с НАСА космического ведомства СССР


Можем, потому что нихуя подобного не утверждалось. Во время высадки температура снятая расположенным на поверхности Луны датчиком EASEP показывала от -23℃ до +7℃
Так что перед тем как делать выводы надо проверить верность предпосылок, иначе будешь выглядеть как тупорылый заговорщик.
Правильно не "они не были на луне иначе бы галоши поплавились", а "правда ли там 120 градусов было, это бы галоши поплавило, кстати были ли галоши?", тогда не стал бы выглядеть шизиком.
image.png438 Кб, 1280x939
114 697434
Помогите понять МАСШТАБ через сравнение.
Вот есть Земля и есть Уран. Если мы натянем карту поверхности глобуса Земли на поверхность Урана с сохранением пропорций, то размером с какие острова на карте Урана будут земные Австралия, Африка, Евразия?
115 697446
>>697434
Рудазов, залогинтесь.
116 697469
>>697434
Ну масштабни площадь материков/островов в 15.86 раз. Из Африки ровно Гренландия получается. Евразия в полтора раза больше. Из Австралии получается примерно так Суматра либо Баффинова Земля.
https://ru.wikipedia.org/wiki/Список_островов_по_площади
117 697487
А что чувствовали астронавты когда ступали на поверхность разогретую до 120 градусов?

Ножки приятно грело?
118 697489

>всерьёз кормить шизика


Никогда такого не было и вот опять
119 697493
Что будет с майонезом если размазать его по корпусу межпланетного корабля? По идее, это идеальный вариант защиты против мелкого мусора. Он просто тупа будет соскальзывать вдоль корпуса и налипать, а при входе в атмосферу получим ещё и термозащиту.
120 697498
>>697045 (OP)
Где тред навозных жуков?
121 697501
>>697493
В вакууме маянезик (как и любая водосодержащая нямка) моментально вскипит и засохнет.
122 697505
>>697501
Бесполезный майонез.

А по кепчуку есть инфа?
123 697509
>>697425
Очень интересно читать эту парашу (нет). Поверхность Луны должна быть теплее поверхности Земли, потому что атмосферы нет, пыль не летает, а значит ничего экранирует солнечные лучи, и ничего не отводит тепло от поверхности. Так же, как и все спутники, мкс и жопы космонавтов, поверхность Луны не имеет возможности рассеивать тепло быстрее, чем идёт нагрев от солнца. И писать, что на Луне 120 градусов - максимум, а -70 - минимум, это как насрать в рот каждому верующему в космос. На Луне климат мягче, чем на Земле. Гениально. Любители верить кушают говно большой ложкой.
124 697511
>>697423
У меня неделю назад в 3 часа дня было полнолуние (что уже доказывает, но верующим не докажешь), и я мог сравнить, что Луна по яркости как оцинкованная крыша.
L2-halo-orbit.png37 Кб, 850x452
125 697516
Есть ли какая-нибудь возможность забросить аппарат в L2 без траты горючки на удержание, или там с микроскопическими затратами?

Какая-нибудь хитровыебанная высокостабильная траектория, чтобы цепляться бровями за гравитационные возмущения Юпитера, или что-нибудь в этом духе.
126 697523
>>697498
>>697045 (OP)
бамп сука куда проебали самый важный тред доски, молокососы?
127 697524
>>697509

> и ничего не отводит тепло от поверхности


Тепло отводится излучением, а поскольку атмосферы пыли и облаков нет, то ничто его не задерживает.
128 697525
>>697511
Как оцинкованная крыша освещенная чем? Свечкой, прожектором, сверхмощным квазаром?
1614729895720.jpg233 Кб, 890x892
129 697539
>>697524
>>697525
Вы чё блять, дебилы? Потом не жалуйтесь.
130 697542
>>697539
Они просрали тред навозных жуков!
131 697558
>>697487
Не, ничего не чувствовали. Им не до того. Их самих охлаждать надо. Когда вы говорите о космонавтах на луне, вам стоит вспомнить зачем у них рундуки. В рундуках их, еще и система охлаждения. Если тебя засунуть в открытый космос в скафандре, то ты без охлаждения сдохнешь не от охлаждения, а от перегрева (от собственного тела).
132 697567
>>697511
Еще раз, чем замеры проводились, где их результаты?
133 697587
>>697523
>>697498
Его стёрли пару лет назад. Именно стёрли
134 697595
>>697241

>Хуйню про увидеть будущее забудь


>>697276

>Нет.


Почему? Ведь доказано же, что вблизи массивных объектов время замедляется для стороннего наблюдателя. Т.е, если атомные часы поставить на Земле, а вторые на МКС, то они покажут разное время, т.к. из-за массы Земли время тут будет течь медленнее для наблюдателя с МКС. Ну так это лёгкая Земля, а какой пиздец творится вблизи Стрелец А, например? Т.е., у поверхности ЧД время должно течь очень медленно для стороннего наблюдателя, тогда как для наблюдателя у ЧД время за его спиной должно течь очень быстро, т.е. подлететь к ЧД - это способ заглянуть в будущее?
135 697605
Какая по сути система координат на Земле? По идее сферическая: широта, долгота и высота над уровнем моря? Если не брать моря, СК сводится к полярной? То есть если взять ноль на северном полюсе, то долгота будет углом радиус-вектора, а широта - расстоянием. А к декартовой как можно свести?
136 697608
>>697605
К декартовой не получится. Точнее возможно задать систему координат, но она будет совершенно неприменима в реальности, разве что для объяснения компу как рендерить что-то.
137 697621
Что будет с биоритмами человека, если его посадить в ракету и отправить далеко в космос, где звезды будут примерно одинаковой яркости? Алсо, что будет с женскими циклами на такой ракете? Ведь они зависят от луны, насколько я знаю...
138 697622
>>697605
В общем смысле сферическая, но Земля это геоид неправильной формы, а форма нужна очень точная, привязка бывает к разным вещам (географические фичи, гравитационное, магнитное поле и т.п.) да и поверхность постоянно меняется. Поэтому датумов много и актуальные постоянно поправляются, например WGS, EGM, WMM, ПЗ, ГСК и т.п.

И это только геоцентрические координаты для геодезии. Для орбитального движения используют самые разные точки отсчёта - поверхностные, барицентрические, привязанные к разным телам и т.п., например J2000, GCRF, норадовская TEME, много их.
139 697623
>>697621
Ничего, достаточно держать режим и соответствующее освещение.
140 697624
>>697622

>Земля это геоид неправильной формы


Скажи честно, ты долбоеб?
15917142033080.jpg12 Кб, 300x150
141 697625
>>697624
Пожуй говна, хуесос. Какого ещё ответа ты ожидаешь?
"Геоид, он неправильной формы", можешь подставить нужное. Только не делай вид что ты не понял о чем речь.
Нет нужды играть в агрошкольника, если ты не хочешь чтобы тебе не срали в горло, мудак.
142 697626
>>697625
Тушись, обиженка.
143 697628
>>697623
Ок. А если в глубоком космосе родятся дети, какие будут биоритмы/женские циклы у них, если они никогда не были на земле?
144 697629
>>697324
>>697324
ты хоть понял чё написал? процесс производства стекла представляешь?
145 697630
>>697621
Все эти биоритмы, это такое. Ничего с женскими циклами не будет, от луны они вряд ли зависят, а нарушения будут хоть так, хоть сяк. Мы (как организьмы) для среды этой не очень подходим (в рамках адекватного).
>>697628
Обычные биоритмы будут (что это вообще за говно). И циклы будут нормальные, если на планете родятся. А в космосе не родятся. То что родится в космосе, можно будет в кунсткамере показывать. И то не факт что сформируется, практики нет, и не предвидится. Рожать детей в космосе, а не на планете, очень плохая идея. И рожать, и зачинать.
146 697635
>>697608
>>697622
Спасибо за ответ. Что значит геоид неправильной формы? Я правильно понял, что геоид - суть обычный правильный шар, только в этой модели ещё учитывается вращение и следовательно сплющивание у полюсов? То есть есть заполненная океаном планета, и водная поверхность под действием вращения меняет форму, но здесь не учитывается солёность, температура, атмосферное давление? А неправильная форма - рельеф материков? Потому что для правильного геоида мы мысленно продлеваем поверхность океана под материками?

Это, если что, этот >>697605 анон отвечает.
147 697640
Расширение вселенной это какая-то жижа и наебалово. Типа вот тут пространство не добавляется, а тут смотрите добавляется. Это никак не вяжется с раздувающимся шариком. Добавляться должно везде или нигде.
148 697641
>>697640
Вот у тебя поверхность воды, из глубины вверх бьёт подводная струя, создавая на поверхности растекание во все стороны одновременно. Кинул кучу шариков - они расплылись в стороны. Связал шарики ниткой - не расплылись, нитка держит. Это не значит что течения не существует
149 697645
Есть ли такая орбита где бы расширение времени от СТО и от ОТО отменяли бы друг друга?
150 697648
>>697641
Ясн
151 697653
>>697045 (OP)

>7777898.png


Это ЧД звёзды пожирает что ли? Насколько это реалистично*
152 697654
>>697653
На 0%. В реальности ЧД будет либо плавно разматывать звезду, высасывая с неё верхние слои и аккрецируя их на себя, если это двойная система и они вращаются друг вокруг друга. Либо нахуй распидорит градиентом гравитации в облако газа, если звезда слишком близко пролетит. https://www.youtube.com/watch?v=ubBzcSD8G8k
153 697655
>>697595

>для наблюдателя у ЧД время за его спиной должно течь очень быстро


Для него все будет совершенно обычно, никакого ускорения времени.
154 697658
>>697640
Антисетипетух, спок.
155 697660
>>697655
Но почему же? Часы на МКС идут быстрее? Идут. Возле чд это будет на порядки больше.
image.png47 Кб, 2027x631
156 697669
>>697660
ОТНОСИТЕЛЬНОСТЬ
Теперь смотри. Откуда мы знаем что на МКС часы идут быстрее? Ну мы сравнили их с земными. Точно так же и здесь. Наблюдатель 1 не замечает что что-то изменилось. Наблюдатель 2 вообще на условной МКС летает.

Так как понять что что-то поменялось? Надо показания часов1 (и календаря1, его не нарисовали, кризис), сравнить с часами 2 и календарем2. И тут мы заметим, что ой, какая разница то сурьёзная. Вот оно ВРЕМЯ и ПРОСТРАНСТВО. Неразделимое увы и ах.

Это первый момент. Т.е. если наблюдатель 1 не имеет земных часов под рукой, то нихера для него не меняется буквально. Второй момент в том, что на восприятие наблюдателя1, могут наложиться некие неизвестные пока эффекты. Но тут пока практикой не проверишь (сомнительная затея, хотя яб записался), не узнаешь. Уот.
мимокста
157 697675
>>697669
>>697660
Ты, не знаю. Интерстеллар смотрел? Ну вот когда они на планету залитую водой высаживались? Если очень грубо говоря, то вот так оно и работает. Прямо как показано. Только там наблюдатель 2 это космоколесо с негром, а наблюдатель 1 это десантники высадившиеся на планету. У них там час прошел, а у него там уже лет 10, или сколько уже, не помню. Но это так, для визуализации пример.
158 697676
>>697669
Я не вижу логики в твоих словах. Мы знаем, что часы идут быстрее, но разницы никакой нет. Как это? Меж Землёй и МКС возможно, но потому что там разница в 1*10^-10500 секунд всего лишь. А Купер 80 лет потерял!!!

>Это первый момент. Т.е. если наблюдатель 1 не имеет земных часов под рукой, то нихера для него не меняется буквально


Ну как, если он возле сверхмассивной чёрной дыры находится и часики тикают быстрее, то он гипотетически должен там увидеть эволюцию Вселенной, её смерть.
9k.png24 Кб, 1611x521
159 697677
>>697645
конешно есть ! я щас её нарисую
160 697678
>>697675

> Интерстеллар смотрел?


Ну да. Там это сила любви. пятимерные шкафы книжные. Очень интересно.
161 697679
>>697678
рядом с чёрной дырой сила любви вытягивается, а шкафы становятся длинней в не то измерениее
162 697680
>>697676
В прямом, разницы никакой нет. Есть еще пример с радиосигналами (будь возможность их досылать от наблюдателя1 к наблюдателю2). Про что мать твою трехсисечную и зеленую (с чешуей) за ногу, теория относительности? Про относительность. А для относительности одного объекта недостаточно. Потому что отношения не возникает. Что относительно чего, если у нас всего 1 наблюдатель с часами?

>А Купер 80 лет потерял!!!


Ты в фильм полностью не упирайся. Они могли и пропорции нарушить, и т.п. Интерстеллар для масс-зрителя доносит общий и поверхностный смысол того, как оно там примерно. И не путайся. Потеряно было ВСЕГО 80 лет. Для их то путешествия это еще не так уж и плохо.

Ты пытаешься отделить время от пространства. А вот этого делать не стоило. Это первое. А второе.

> он гипотетически должен там увидеть эволюцию Вселенной, её смерть.


Ты понимаешь что такое черная дыра? Это не окно в Европу (или в куда угодно). В ней нет вселенной. В ней не будет даже целого тебя, спустя не такой уж и большой промежуток времени (для тебя). Это к вопросу о том, как летать через черные дыры на планеты миллер, которые хер знает где. С этим серьезные траблы, ведь пока все намекает на то, что ты даже в виде частиц никуда не долетишь, не то что в виде корабля с человеками.
>>697678
Трагедия интерстеллара в том, что они создали очередную манямечту. К которой есть некий скептицизм. Опровергнуть этот скептицизм пока нечем и никак, и вы не спешите туда нырять в своих головах. А то ведб и не вынырнете, и обратки никакой не дадите. И сиди потом думай, вас там спатегировало, или вы планеты миллер покоряете.
163 697681
>>697658
Это что такое? Не гуглится даже.
164 697683
>>697676
Не знаю, понял ли ты что, пытался попроще объяснить. Если ничего понятнее не стало. То гугли:
"Гравитационное замедление времени".
Мб хоть так разберешься.
>>697681
Сети-петухи: воюют за поиск инопланетных цивилизаций радивом.
Антисети-петухи: воюют против Сети-петухов, которые воюют за поиск инопланетных цивилизаций радивом. В SCI в корне борды, даже правило есть на этот счет.
165 697685
>>697680

>Про что мать твою трехсисечную и зеленую (с чешуей) за ногу, теория относительности? Про относительность. А для относительности одного объекта недостаточно. Потому что отношения не возникает. Что относительно чего, если у нас всего 1 наблюдатель с часами?


Отношение между человеком у ЧД и остальной Вселенной, вторую точку с часами можно взять произвольно, пусть это будет Энцелад.

>Потеряно было ВСЕГО 80 лет. Для их то путешествия это еще не так уж и плохо.


Не спорю, но можно вообразить ЧД, из-за которой пройдёт 1000 лет, а из ОТО вывести такую, где пройдёт миллиард лет. Вот тебе и будущее.

>Ты понимаешь что такое черная дыра? Это не окно в Европу (или в куда угодно). В ней нет вселенной


Здесь проблема была лишь в грамматике "там" - не наречие, а просто как мусорное слово-паразит. Я говорил про Вселенную не в ЧД, а обычную Вселенную нашу. То есть ты плывешь спиной к ЧД, а наблюдаешь остальную Вселенную.

>>697676
Если так, то укажи на ошибки в моих рассуждениях.
166 697688
>>697685
Ага, только 2 занятных эффекта, не позволят тебе промотанные 1000 лет заметить. Потому что они промотались где-то там, а не у тебя тута здеся. Это можно было бы исправить, моги мы от условного энцелада с тобой по "радиву" общаться. Правда ты бы все равно затроил, и BSODнул бы головой. А еще за косарь лет, могло столько всякого произойти, что с тобой уже и через 500 некому связаться.

>вообразить ЧД


С воображением всего этого дела вообще дело худо. Поэтому и приходится к научному методу возвращаться. Без него, мы в дурку поедем всем спейсачом (некоторые уже оттуда капчуют).

>То есть ты плывешь спиной к ЧД, а наблюдаешь остальную Вселенную.


Ясно, тогда солидарен вот с этим >>697276
По части:

>фотоны из внешней Вселенной благополучно будут попадать тебе на глаза через горизонт. Именно поэтому ты даже не заметишь, что оказался за горизонтом, т.к. это не физическая поверхность.



Правда это тоже гипотетически.
167 697692
Какова вероятность существования планеты, где есть океан или хотя бы море какавы?
168 697697
>>697688

>Правда ты бы все равно затроил, и BSODнул бы головой.


Что это значит? Можно "растянутый" сигнал тоже правильно закодировать кампуктером жеж.

>С воображением всего этого дела вообще дело худо. Поэтому и приходится к научному методу возвращаться. Без него, мы в дурку поедем всем спейсачом (некоторые уже оттуда капчуют).


Главное, чтобы парадигме и ОТО не противоречило.
169 697698
>>697660

>Часы на МКС идут быстрее? Идут


Это относительный эффект, который виден лишь при сравнении показаний одних приборов с показаниями других. Для самих наблюдателей в своей системе отсчета ничего не меняется, для них время течет как обычно.
170 697699
Где какава быстрее остынет, на земле или на мкс?
171 697700
>>697697
Ну я тебе говорю, Тайрон, нигга, ты пока там куковал, у нас тут 10 цивилизаций сменилось, и 4 войны мировых. А и да, земля все. А еще мы не на энцеладе и не на земле сидим, а вообще на глизе с другой системы капчуем. Ты скажи тайрон спасибо, что мы вообще про тебя не забыли со всем этим говном за 1000 лет.
А ты такой: ...
Это я еще другие "красные смещения" не беру. Например, языковое. Так то 1000 лет это нехилый предел, создающий кучу трудностей. Даже на уровне диванных размышлений.

>Главное, чтобы парадигме и ОТО не противоречило.


Ну эт да.
>>697692
На уровне погрешности, но не нулевая. Заяц с банок несквика ты? Узнал тебя по твоим шизоидным высерам.
172 697793
Мб достойно отдельного треда. Учился бы на астрофаке, сделал бы обширный диссер.
Короче, обсудим кратер на Юкатане, его удар и великой вымирание динозавров? Мб документалки есть? Подробные научные исследования?

Это правда, что один астероид размером 10км поднял пыль, которая закрыла все небеса(кроме полюсов? мб есть симуляции/модели?) на пару лет, приостановила фотосинтез всех растений и послужила причиной исчезноваения пищи для динозавров? Всего 2-3 года ядерной зимы без солнца - и всё?

Это был один астероид? Мб был другой +- в то время(сейчас скрытый под землей, хотя глубокий и широкий должен быть выден), и ядерная зима длилась не 2-3 года, а дольше?
173 697795
Блядь, а прикиньте, стоит ведь только обосноваться на Титане и можно по всей СС на химических двигателях колесить во все стороны, охуенно же!
ыгт-ир.png108 Кб, 1863x1111
174 697805
>>697654
а вот если звезда-пизда прям на чд летит, то наверно и никакого диска не будет, она звезду сразу всосёт или подавится? тогда будет большой бабах.. кстате там не блекфейс
175 697806
>>697645
3000 километров
176 697807
Geoid.png31 Кб, 500x190
177 697821
>>697635

>Я правильно понял, что геоид - суть обычный правильный шар, только в этой модели ещё учитывается вращение и следовательно сплющивание у полюсов



Неправильно, то что ты описываешь, называется земной эллипсоид. Геоид учитывает ещё и неравномерное распределение масс внутри Земли, в т.ч. и выше уровня моря. Поверхность геоида мысленно продолжается под материками и горами, но отклоняется там вверх и вниз от идеального эллипсоида.
178 697822
>>697045 (OP)
Господа, напомните, почему нельзя поддерживать орбиту скажем ГСОшных спутников солнечным парусом?
Стикер255 Кб, 500x500
179 697823
>>697822
Потому что если на ГСО откроешь парус, тебя сдует с нее к хуям.
ГСО охуенно стабильная орбита, там проблем с поддержанием нет.
Вот для свода может быть и можно было бы парусом подруливать, на солярке бы сэкономили.
180 697824
>>697823
Если прямо сдуть может, то и корректировать бы изи. Там по массе того спутнтика половина - топляк, орбиту надо поддерживать, после чего выводить.
И да, хуле бы не сводить тогда с орбиты?
181 697828
>>697805
>>697677
Проиграл с мэдскиллза чёт.

Напрямую - всосёт сразу наверно. Но надо понимать одну вещь - ЧД намного меньше звёзд по размерам. ЧД размером с типичную звезду, а тем более больше её - это гигантская, сверхмассивная йоба, обычно такие только в центре галактик бывают. И всё ближайшее окружение крутится боком вокруг неё, неоткуда взяться прямому столкновению, практически всегда будет мимопролёт. Для прямого нужны какие-то реально экзотические условия. Например взрыв сверхновой, запульнувший остаток куда-то, а тот в свою очередь пролетел мимо анрилейтед звезды и снайперски столкнул её гравитацией в ЧД. Вероятность такой хуйни просто пиздец маленькая. Или столкновение галактик, которые обычно при столкновении просто перемешиваются - реального столкновения объектов почти никогда не происходит, а уж тем более прямого.
182 697829
>>697823
ГСО это как раз нестабильная орбита. На ней есть две квазистабильных точки, к которым дрейфует всё что умирает без топляка. Но и с них всё укатывается наружу и прецессирует. Поэтому и сводить с неё не надо.

>>697822
Не уверен что это возможно с парусом разумных размеров. На высотах ГСО парус уже имеет смысл, но с другой стороны ГСОшные йобы все тяжёлые и большие (в них напихивают дохуя, ибо слоты на ГСО ограничены разрешением клиентских терминалов, т.е. размером наземных тарелок). Плюс ГСОшный спутник залочен ориентацией на землю, а не солнце, и у большинства и так уже есть панели с одной степенью свободы. Парус это ещё один кусок механики, которой в космосе следует избегать, и не должен при этом заслонять ничего.
В целом это будет зависеть от силы возмущений и требуемой ориентации паруса, сколько его крутить, будет ли заслонять транспондеры/оптику/батареи и т.п. Тут надо засовывать в симулятор вроде GMAT и думать, сходу не скажешь. Скорее всего нет.
Но самое главное это не особо нужно. Затраты на поддержание ГСО это до 53м/с в год, в зависимости от градуса. Это не настолько много, чтобы парус городить.
183 697830
>>697828
у чд невьебенная масса, такшто прямые столкноения палюбас происходят
184 697831
>>697830
Неа. Схуяли им происходить? Обычная ЧД ничем по поведению от звезды той же массы не отличается. Много прямых столкновений звёзд наблюдается? Жирная ЧД сидит жопой в центре галактики близ барицентра оной или прямо на нём, т.е. вокруг неё всё крутится. Просто нет объектов, которые бы вдруг внезапно решили лететь на неё. А если появятся случайно - они почти всегда чуть-чуть промахнутся, ибо ЧД маленькая относительно своей массы. А те звёзды поблизости неё что постепенно отклоняют траекторию поближе - тем более напрямую не хуйнут.
185 697833
>>697829

>ГСО это как раз нестабильная орбита. На ней есть две квазистабильных точки, к которым дрейфует всё что умирает без топляка. Но и с них всё укатывается наружу и прецессирует. Поэтому и сводить с неё не надо.


Ты с лагранжем путаешь?
186 697837
Уэббисты-ракетчики, можно блиц-ответ на блиц-вопрос?
Мы в Л2 летать на починку не можем потому что слишком далеко, дорого и рискованно, или есть какая-то фундаментальная проблема?
187 697838
>>697837

>слишком далеко, дорого и рискованно


Да.
188 697839
>>697838
Понел, спасибо. Звучит обнадёживающе.
189 697861
>>697837
1. Потому что нет подходящих средств для полёта туда и починки. Разве что Орион с гипотетическим модулем-доком для ВКД и обслуживания аппаратов, но Орион даже сегодня не готов и неизвестно ли будет готов, как и SLS во втором более тяжёлом варианте. Удалённое управление беспилотным аппаратом же там здорово затруднено, т.к. дотуда 1.5 млн километров и задержка сигнала не позволит. Т.е. миссия должна быть пилотируемой или нереалистично автоматизированной.

2. Орбита JWST нестабильная, ему надо постоянно тратить топливо на её поддержание. Поэтому он проживёт всего около 5 лет, это не долговременный телескоп как HST.

Ввиду этих двух вещей бессмысленно предусматривать обслуживание телескопа.

Фундаментальной проблемы долететь в L2 на пилотируемом аппарате - конечно никакой нет. Есть некоторая проблема конкретно с JWST, это его хрупкость. Это ёбаный пятислойный парник и супер тонкий инструмент, чуть что тронешь и всё пойдёт по пизде.

А так-то заправка через сопло беспилотным аппаратом уже была опробована на низкой орбите. Может и с JWST так решат поступить, продлив ему жизнь. Но вряд ли.
190 697862
>>697833
Не путаю. ГСО нестабильна.
191 697879
>>697861
Я тебя понял. Значит технически и теоретически это возможно. Уже радует. Значит перспектива на продолжение JWST (уже обслуживаемых) есть. Я тебя понял, спасибо за развернутый ответ.
192 698199
Сколько по-времени звёзды взрываются? Пишут мол бетельгейзе может ебануть, сколько она будет светится в небе? Час? Месяц? День? Год?
193 698203
>>698199
Сам взрыв длится минуту-другую в зависимости от типа сверхновой. Но светимость может часами увеличенная быть, чисто физически объект огромный. А потом говна разлетаются, на это нужны дни-месяцы-годы-столетия.
194 698215
Почему JWST не запустили на примерно такую же орбиту (круговую, околоземную), как и Хаббл? Ну чтобы можно было его перезаправлять и чтобы он тоже проработал 20+ лет?

Мне кажется нелепым, что такой дорогущий долгострой и крутейший инструмент проработает всего 5 лет и обякнет, жидко пукнув хладагентом
195 698217
>>698203

>А потом говна разлетаются, на это нужны дни-месяцы-годы-столетия.


https://en.wikipedia.org/wiki/Supernova_remnant
Вот такие хуйни считай перманентно висят в небе. Как говорится, куда ты сбежишь с подводной лодки, космос слишком большой чтобы его заполнить.
196 698218
>>698215
Чтобы смотреть теплощитом на Солнце всегда, закрываясь от него. При этом чтобы болтающаяся рядом земляха не перекрывала бы обзор и не мешала бы пилить ебовейшие выдержки.
197 698219
>>698215

>жидко пукнув хладагентом


Как раз хладагент там бесконечный, точнее замкнутая система и может бесконечно морозить.
198 698220
>>698218
А хабблу будет прямая функциональная замена?
199 698222
>>698220
Не совсем. JWST почти не оптический телескоп, он больше инфракрасный. Оптическим задуман LUVOIR, по той же схеме, он вообще мультиспектральный. Но он хуй знает когда полетит, если вообще полетит.
200 698234
Сейчас запустили Уэбба, чтобы он смотрел вдаль и в раннюю Вселенную. Отсюда вопрос: мы знаем, что Вселенная бесконечна и у нее нет какого-либо центра. Но, вот Уэбб будет смотреть на объекты в 13 млрд св. лет. Соответственно там эти объекты находились 13 млрд лет назад. А если он посмотрит на 13 млрд св. лет в противоположном направлении? Увидит ли он там что-либо? И если да, то как так получится? А если нет, то не будет ли это означать, что мы находимся в какой-то стороне от бывшей сингулярности, т.е. "центра Вселенной"?

И еще вопрос из то же оперы. Предположим, у нас есть объект, который имеет собственную скорость по отношению к нам. В какой-то момент он испускает свет, который идет к нам под углом альфа. Но под воздействием расширения Вселенной этот объект "перемещается", скажем, в сторону от испущенного луча света и испускает новый свет под углом бета. Т.о., возможна ли ситуация, когда мы за счет исключительно такого расширительного дрейфа можем наблюдать один и тот же объект но в разных направлениях?

Понимаю, что вопросы сформулированы путано, но надеюсь вы поняли.
201 698241
>>698234

>А если он посмотрит на 13 млрд св. лет в противоположном направлении?


Не понял, в будущее что ли? Для этого машина времени нужна, а не телескоп.
Если ты имеешь в виду просто развернуться на 180 градусов - будет то же самое, но в другой стороне. Расширение пространства везде одинаково, всё тупо удаляется от нас. "Посмотреть на ранние объекты" означает всего лишь увидеть те объекты, которые от нас в 13 млрд световых лет, они все в инфракрасном диапазоне и очень тусклые, поэтому и нужен такой космический ёба-телескоп, чтобы их засечь.

По второму вопросу: нет, расширение сверхсветовое, объект не может опередить его, приближаясь к нам быстрее.
202 698242
>>697831
даж в такую шелупонь как наша ёбаная луна попадают, а чд тащемта потяжелее будет. кстате я старался часа полтора рисовал, хуясе медскилз
203 698245
>>698234

>А если нет, то не будет ли это означать, что мы находимся в какой-то стороне от бывшей сингулярности, т.е. "центра Вселенной"?


У расширения нет центра, как нет центра у поверхности раздувающегося шарика. Бывшая сингулярность это вся Вселенная и есть.
204 698249
>>698234

> А если он посмотрит на 13 млрд св. лет в противоположном направлении? Увидит ли он там что-либо?


Увидит то же самое, но с другой стороны. Первые галактики, но не те что справа, а те что слева.
На подумоть: https://zen.yandex.ru/media/nakedscience/rasshiriaetsia-li-vselennaia-bystree-skorosti-sveta-60109935906c515f4c985d67
(сам чуть не ебанулся пока читал это, сложна)

> Но под воздействием расширения Вселенной этот объект "перемещается", скажем, в сторону от испущенного луча света и испускает новый свет под углом бета.


Так расширение же равномерно в пространстве, разве нет?
205 698258
>>698241
А расширение всегда направлено строго от нас и никогда от нас и в сторону?
>>698245
Тут проблема в том,что у шарика-то центр есть. Ясно,что это некорректная аналогия, но на бытовом уровне оно воспринимается как-то так.
>>698249
Благодарю за ссылку. Но пока уточню, если он в противоположной стороне не увидит ничего - это можно трактовать, как то, что там ничего никогда и не было и вообще там какая-нибудь ударная волна Вселенной и есть?
206 698261
>>698258

>А расширение всегда направлено строго от нас и никогда от нас и в сторону?


Да, строго от нас. Есть небольшая неравномерность, обусловленная флуктуациями ранней вселенной, но она измеряется хуем с маслом, надо с лупой отличия искать.

>Тут проблема в том,что у шарика-то центр есть.


У двумерной поверхности шарика нет центра.
207 698268
>>698258

> Тут проблема в том,что у шарика-то центр есть. Ясно,что это некорректная аналогия, но на бытовом уровне оно воспринимается как-то так.


И что такое центр шарика? У него есть центр тяжести, центр координат, центр окружности(хз как сформулировать правильнее, центр выпуклости иначе говоря), это все разные определения и разные места. А какого-то главного центра нет.
Ну и конечно это аналогия на пальцах, чтобы понять её надо выходить за пределы бытового уровня, иначе не поймешь.
208 698269
>>698258

> если он в противоположной стороне не увидит ничего - это можно трактовать, как то, что там ничего никогда и не было и вообще там какая-нибудь ударная волна Вселенной и есть?


Можно, но он скорее всего увидит, можно анус ставить спокойно. Какая такая ударная волна, на основе каких предположений ты это вывел?
209 698280
Если Вселенная не бесконечна и если она плоская или гиперболичная, то как может выглядеть момент подхода человека к её краю?
210 698296
Чего после джеймса уэбба ждать? Очевидно, что он какая-то хуйня без задач и ничего прорывного в нём нет. ELT вроде в обозримом будущем появится, а что с LUVOIR? Есть ещё какие-нибудь проекты?
211 698300
>>698296
Из телескопов: тридцатиметровый, гигантский Магелланов.
Из космических: Спектр-УФ и Миллеметрон.
212 698305
>>698300

> Миллиметрон.


Слоуфикс.
213 698307
>>698280

>то как может выглядеть момент подхода человека к её краю?


Никак. Фазовая скорость расширения вселенной - 68 км/(c⋅Мпк). "Край" (если он вообще есть) вселенной (который на расстоянии в 14000 Мпк) удаляется от нас втрое быстрее скорости света.
214 698335
>>698307
Ну представим что человек двигается вместе с краем, вместе с пространством у сасого краешка. что там за физика на краю? законы плавно отменяются когда переходишь вне пространстаенную расаеянную грань или грань как обрыв: вот тут времяпространство, а там уже нет нихуя.
А какого это последнему кксочку пространства время до рагиваться до нихуя? Какого это когда справа от тебя пространство время а слева нихуя? наверняка на тебя действует какая то сила. Вталкивающая или выталкивающая? Больно касаться нижуя? Или это типо как стенка, типо просто там нихуя?
А как выглядит нихуя?
Нихуя выглядит как нихуя.
215 698336
>>698307
Ну представим что человек двигается вместе с краем, вместе с пространством у сасого краешка. что там за физика на краю? законы плавно отменяются когда переходишь вне пространстаенную расаеянную грань или грань как обрыв: вот тут времяпространство, а там уже нет нихуя.
А какого это последнему кксочку пространства время до рагиваться до нихуя? Какого это когда справа от тебя пространство время а слева нихуя? наверняка на тебя действует какая то сила. Вталкивающая или выталкивающая? Больно касаться нижуя? Или это типо как стенка, типо просто там нихуя?
А как выглядит нихуя?
Нихуя выглядит как нихуя.

Вот бы нихуя увидеть одним хоть глазочком.
Вот бы увидеть нихуя? Ты долбоеб?
216 698340
>>698234
Тебе в сциентач надо, местные аноны как правило как и ты не понимают не только ОТО, но и СТО.

> мы знаем, что Вселенная бесконечна


Этого мы не знаем.

>у нее нет какого-либо центра


Это мы знаем.

> Но, вот Уэбб будет смотреть на объекты в 13 млрд св. лет. Соответственно там эти объекты находились 13 млрд лет назад.


13 млрд лет назад звёзд ещё не было, не то что планет, но это не существенно, пусть будет 13

Вот это утверждение — это суржик из физик Ньютона и Эйнштейна. Тебе, чтобы это всё понять, надо сдвинуть парадигму. Прочитай внимательно: никакого абсолютного времени нет. Свет от этих объектов действительно летел к нам 13 млрд лет в нашей системе отсчёта, но это не даёт нам возможности говорить, что эти объекты переехали к моменту СЕЙЧАС, потому что никакого общего СЕЙЧАС у нас и этих объектов нет. Лучше воспринимать в качестве СЕЙЧАС как раз то, какая информация к нам о них поступает в реальном мире.

> А если он посмотрит на 13 млрд св. лет в противоположном направлении?


Там будет примерно то же самое, что и в любом направлении.

> Предположим, у нас есть объект, который имеет собственную скорость по отношению к нам. В какой-то момент он испускает свет, который идет к нам под углом альфа. Но под воздействием расширения Вселенной этот объект "перемещается", скажем, в сторону от испущенного луча света и испускает новый свет под углом бета.


> Т.о., возможна ли ситуация, когда мы за счет исключительно такого расширительного дрейфа можем наблюдать один и тот же объект но в разных направлениях?


См. https://ru.wikipedia.org/wiki/Лоренцево_сокращение
Эти объекты, которые мы наблюдаем, как правило светятся не как маяк, а постоянно, поэтому изображение объекта будет непрерывным.
Правда есть вот такой интересный эффект https://ru.wikipedia.org/wiki/Гравитационная_линза , но к перемещению объекта это отношения не имеет.
217 698341
>>698340
фикс

>какая информация к нам о них поступает в реальном мире времени

218 698343
>>698280
А если вселенная в форме чайника, то какой чай в неё налит?

Нет вообще никаких внятных гипотез про край вселенной, поэтому считается, что она либо бесконечная, либо замкнутая.
Стикер63 Кб, 468x467
219 698345
>>698222

>Но он хуй знает когда полетит, если вообще полетит.

220 698348
>>698343
Заваренный на жидком вакууме, очевидно же!
221 698360
>>698234
13 миллиардов лет назад все объекты вселенной были +- в одном месте, она была маленькой, это сейчас она ебейших масштабов, так что куда бы ты не посмотрел, увидишь центр древней вселенной.
222 698386
>>698343

>то какой чай в неё налит?


Earl Grey
223 698392
>>698199

>сколько она будет светится в небе?


Может светиться месяцы.
224 698393
>>698234

>что Вселенная бесконечна и у нее нет какого-либо центра


Центра нет, но она не бесконечна.

>А если он посмотрит на 13 млрд св. лет в противоположном направлении?


Видимая часть Вселенной больше десяти гигапарсек в размере, это больше 13 млрд св. лет. И на таком расстоянии видны разные объекты. В каком бы направлении он не посмотрел, он увидит то же самое. Вселенная на таких масштабах однородная и изотропная.

>А если нет, то не будет ли это означать, что мы находимся в какой-то стороне от бывшей сингулярности, т.е. "центра Вселенной"?


У Вселенной нет центра, ты же сам написал. Сингулярность - это не место, это состояние, в котором находилась ранняя Вселенная.
225 698394
>>698280
У нее нет края.
226 698405
Хочу увидеть момент до времени, момент до БВ.
227 698431
>>698360

>куда бы ты не посмотрел, увидишь центр древней вселенной.


Получается, что центр на краю, снаружи Вселенной?
228 698438
>>697045 (OP)
А както наука описывает состояние вещества в черной дыре?
ок мы знаем что можно все сдавить до уровня решетки протонов и нейтронов. А если давить еще дальше, то что там будует?
В центре черной дфры кваркоглюонная плазма как в бв?
Типо вещество сдавливается, но до какого то предела же, а потом ее удерживает от дальнейшего сжимания?
229 698440
>>698438
Удивляет чего только про черные дыры не напридумывали, а там на самом деле в центре просто кусок кварков сжатый до пиздец каких размеров, что выдавило струны наружу и гравитация сильная итд, но нет этих ваших прорезаний пространства?
с чего вообще ктото решил, что пространство можно протыкать, время остаравливается итд. хуйня же, нет?
1426186026ekzoplaneta-alfa-centavra-b-b.jpg60 Кб, 1024x590
230 698448
Анонасы, какой нужен телескоп что бы увидить воочию как на пике экзопланету. Чрезвычайно большой телескоп, сможет хотябы в в 16x16 пикселей увидеть к примеру proxima b?
Если нет, то какое должно быть зеркало что бы рассмотреть так подробно близкие экзопланеты?
231 698449
>>698448
Размером с орбиту Нептуна.
232 698451
>>698405
Там нечего смотреть.
233 698459
>>698438

>А както наука описывает состояние вещества в черной дыре?


Никак, никто не умеет еще описывать то, что происходит в центре черной дыры, в сингулярности. Для этого нужна квантовая теория гравитации, а ее нет и неизвестно, когда появится. Скорее всего там планковская плотность, а при такой плотности никакие частицы уже существовать не могут (по крайней мере известные нам), не может существовать даже классическое пространство. Пространство там искривляется настолько сильно, что как бы сворачивается, и такие понятия как расстояние и длина теряют всякий смысл.
234 698487
Анончик, помоги.
Нужно узнать в скольки орбитальных плоскостях запущены спутники фирмы ICEYE. Интернет молчит по этому поводу.
235 698539
>>698459
Ну сворачиваются и свобрачиваются, не протыкаются же.
А с чего кто то считает, что там квантовская плотность? С чего взяли, что в центре должна быть обязательно сингулярность? Звучит как изначально хуевое предпопложение на самом деле
236 698540
>>698539

>квантовская плотность


планковская плотность
237 698545
>>698539
1) Нет никаких известных сил, способных остановить дальнейший коллапс до сингулярности, гравитация пересиливает их все.
2) Под горизонтом событий пространство настолько искривлено, что взаимодействия не распространяются наружу от центра. Даже если бы отталкивающие силы существовали, они бы попросту не могли достигнуть частиц, лежащих выше, и остановить их падение в центральную точку или кольцо сингулярности.
238 698553
>>698449
>>698448
Нет. Достаточно размером с диск солнца, и будет размер 50-100 метров\пиксель.
https://www.youtube.com/watch?v=X1LzhFikoqE
В принципе возможно в ближайшие 100-200 лет.
239 698556
>>697156
Когда нейтронная звезда появляется у неё овердохуя энергии что-то порядков 10 в 52 эрг (если не путаю) и в ближайшие дни или столетия(в зависимости от массы) она остывает до определённого уровня, но потом выходит на плато и остывает овердохуя лет.

С нейтронной звезды не перекачаешь вещество в обычную звезду. Скорее наоборот случится, что бы перекачать из нейтронной звезды надо в двойную систему поместить ЧД. Но тогда ты это вещество не получишь, т.к. оно упадёт за горизонт.
240 698558
Поясните за инструмент "Прибор для работы в среднем диапазоне инфракрасного излучения" Телескопа JWST.
Он тупо детектирует всё излучение как один цвет или у него можно различить 28мкм от 27 мкм? Насколько он чувствителен в этом плане?
241 698565
>>698553
Что то туманно с этим математическим объединением пикселей, небось дорисовывать будут для той общей картины которую захотят увидеть
242 698568
>>697045 (OP)
Я не верю в динозавров и то что тут было так много жизни до нас.

Наука говорит, что динозавры существовали 200-65 млн лет назад. это 135 000 000 лет. \Динозавры существоали в 2 раза дольше, чем прошло времени от их вымирания и зарождением человечества.

Возьмем 10 000 лет как временной отрезок. Вся более-менее разумная человеческая цивилизация умещается в 10 000 лет.
А динозавры жили 135 000 000 лет. Это 10 000 по 10 000 лет плюс 3500 лет по 10 000 раз. ОХУЕТЬ

Почему крокодилы не изменилсь за, как говорят, 200-100 млн лет? Потому что - я вычитал в комментах ютуба где чел спросил почему они не жволюционировали дальше - главной и едингственной целью эволюции является ВЫЖИВАНИЕ и если форма/модель крокодилов в данной среде(земля) дают им выживание, нахуя им большой мозг и 10 гибких пальцев? Нахуя им С++ и лахерная коррекция зреняия?

Интеренсо. Увидим черз супер-телексом-ракеты поврехност экзопланеты далекой. И по ней бродят динозарвы(или там иные физич...... биолгичессккие принципы постеония жизни воможно?) - и мы буде наблюдать. Пройжет 10 000 лет - все еще они. 100 000 лет - они же. 10 раз по 100 000 лет - они. и так.... миллионы лет. О ХУ ЕТЬ
243 698570
>>698568
Петрович, занюхивай хоть, заебал.
244 698592
>>698558
Ты имеешь в виду MIRI? Это комплексный инструмент. Там есть 4-канальный спектрограф со сменными фильтрами и диффракционными решётками для специализированных задач, какой поставишь, такую длину волны и сфоткает (узкую полоску спектра). И есть обычная монохромная камера, которая сразу весь диапазон воспринимает.
http://ircamera.as.arizona.edu/MIRI/instrument.htm
245 698639
>>698545

>1) Нет никаких известных сил, способных остановить дальнейший коллапс до сингулярности, гравитация пересиливает их все.


Ну а что разве если ты будешь пытаться поместить барионы в одну точку, то они не будут этому препядствовать. Сильное, слабое ... похуй на эти заветы, это более верхний уровень, а тут причине прост глубже.
Но зачем же творить эту шизу непроверяемую, якобы тым понятие длинны пропадает, и коко причинность, и можно увидеть своё будущее, что это нахуй за фэнтези вместо настоящей серьёзной науки?
246 698642
>>698565

>небось дорисовывать будут


Зачем?
247 698643
>>698592
А как он фоткает чтоб цвета разные получались? Кучу раз ставит фильтры на разные длинны волн или сразу большой кусок диапазона, где уже видно, где водород, где гелий?
248 698659
>>698280
Вселенная бесконечная, но не безразмерная.
У поверхности шара тоже нет какого-либо конца и края. Но у шара есть размер.
249 698660
>>698568
Чел ты охуеешь, но почти поллярда лет назад на планете господствовала вендская биота с примитивной симметрией со смещением, всякими пятиконечными симметриями. Ничего подобного сегодня не существует, т.е. все потомки этих существ вымерли, предки нынешних видимо были какими-то другими. Потом была эдиакара с охуевшими креветками и белемнитами размером с метр, потом трилобиты, которые были так разнообразны, что один похож на камень, а другой на залупоглазого покемона. И весь этот зоопарк должен сменять тысячи и тысячи поколений чтобы эволюционировать во что-то другое или вообще вымереть нахуй без следа, дав дорогу другим тварям. И все это за пол лярда лет, что вполне сравнимо с образованием звёзд, планет, да и вообще вселенной. Поэтому меня лично вообще не удивляет, что мы не видим вокруг инопланетян, ведь нам с нашей системой понадобилось 4 млрд лет чтобы появиться, так схуя ли другим потенциальным разумных организмам должно быть надо меньше времени? Да и чем дальше от нас, тем дольше должна информация до нас лететь, а там глядишь уже счёт пойдет на тысячи и миллионы лет, чтобы информация дошла, а вокруг слишком маловероятно что рулеточка крутанулась так же успешно как у нас со всеми нужными переменными.
250 698662
Как можно хакать струны и поля, чтобы в нашей 4-мерной свертке было заебись, там всякие сверхсветовые перемещения, бесконечная материя и энергия, управление временем? Кто-нибудь уже что-нибудь придумал? Ведь не может теория только пытаться описать, ее по-любому можно как-то на практике применять.
251 698664
>>698662
Спасибо за ваш пост, завтра скажу начальнику чтобы начали уже применять смещение времени.
252 698668
>>698487
40 секунд в гугле
https://www.iceye.com/sar-data/orbits
Дальше сам.
>>698568

>200-65 млн лет назад. это 135 000 000 лет


Ага, ясно.
253 698674
>>698459

> Скорее всего там планковская плотность, а при такой плотности никакие частицы уже существовать не могут (по крайней мере известные нам), не может существовать даже классическое пространство. Пространство там искривляется настолько сильно, что как бы сворачивается, и такие понятия как расстояние и длина теряют всякий смысл.


Не проблема, в говне моченые придумают какую нибудь очкошную частицу и все сразу станет понятно
254 698677
>>698539

>С чего взяли, что в центре должна быть обязательно сингулярность?


С того, что коллапс ядра звезды при образовании черной дыры идет до конца и может остановиться только на планковской плотности как на максимальной - сжать что-то до большей плотности невозможно, а на меньшей коллапс не останавливается.
255 698681
>>698677

> С того, что коллапс ядра звезды при образовании черной дыры идет до конца и может остановиться только на планковской плотности как на максимальной - сжать что-то до большей плотности невозможно, а на меньшей коллапс не останавливается.


Это ты так, почувствовал при прочтении научной религиозной литературы?
256 698682
>>698659
У тебя терминология сломалась.
Шар не бесконечный, потому что у него есть конечный объем и площадь поверхности. При этом у поверхности шара нет границы, он замкнутый.
257 698683
>>698643

>Кучу раз ставит фильтры на разные длинны волн


Это. Одновременно 4 канала, прям полный спектр получается длинными выдержками. MIRI ещё и не единственный инструмент. Это телескоп-швейцарский нож.
Но если ты ожидаешь что он будет снимать "как видит глаз, если ему поставить выдержку дохулион часов", не ожидай конечно. Даже если бы он был полностью оптического диапазона - всё равно из-за фильтров разные фичи (водород, кислород и тп.) будут превалировать над реальной видимостью в сложенных картинках.
258 698694
>>698448

>Чрезвычайно большой телескоп


К сожалению этот телескоп недостаточно большой. Нужен по крайней мере Сокрушительно гигантский телескоп, и лишь Труба кошмаров позволит разглядеть в подробностях.
259 698695
Почему джеймса вебба строили 25 лет? Китайцы за пол года целые города отстраивают. А тут какой-то телескоп не особо то большого размера аж 25 лет.
260 698701
>>698695
Был нужен пиздатый криокулер, пиздатый солнечный зонтик, пиздатейший детектор, пиздатые управляемые диффракционные решётки, и самое охуенно пиздатое главное зеркало какое только можно сделать. Всё это потребовало нетривиальных решений и на момент начала разработки вообще не было очевидно, что нужных параметров удастся добиться. Например диффракторы зафейлили и переделывали. Это по чисто телескопной части.

Но большая часть хтоничности - следствие сторонних факторов и всовывания круглого в квадратное:
- попытки облегчить аппарат (не было сверхтяжёлой ракеты)
- попытки уменьшить аппарат, сделав его складным (не было ракеты большого диаметра, хотя вон боенг предлагал запилить нереально здоровенный фейринг специально под это дело)
- заработать всё должно с первого раза, т.к. нет возможности послать пилотируемую экспедицию на починку (не было и сейчас нет пилотируемого корабля с доком), либо беспилотный аппарат. Например система изгибания зеркал сделана именно из-за боязни проебаться с их обработкой.
Всё это вызвало лютейший оверинжиниринг, дало рост затрат во много раз, и задержку в дохуя лет. Кроме того, фирменное насовское сползание сроков/затрат дало рост ещё в 2 раза от ожидаемых.
Если бы не это всё - телескоп сделали бы, испытали и запустили где-нибудь максимум во второй половине нулевых, без особой помпы. Даже если бы к началу разработки был бы хотя бы гарантированный фалкон хэви (к примеру), затрат было бы в разы меньше, и задержек.

С другой стороны, теперь они умеют делать сегментированные космические зеркала, хоть и нереально дорого. В теории могут собирать телескопы, превышающие размерами диаметр любой ракеты. (на практике конечно же этого не будет, если не упростят эти непрактичные способы)
261 698702
>>698682
А почему все против края, что придумывают шар?
Край тоже збс же
262 698703
>>698701
Ну и да, на сегодня доступны куда более точные/чувствительные компоненты, чем применены в веббе. Некоторые компоненты там реально 20-летней давности, детекторы построены по процессу начала нулевых например, а вариант жаваскрипта, зашитый в прошивку, был сделан ДО принятия ECMAScript, и даже компания которая его поддерживала, разорилась 14 лет назад. Реально артефакт другой эпохи.
В этом JWST напоминает ITER немного - пока ITER тянули, изобрели новые сверхпроводники и теперь вообще не факт, что его не обгонят более новые лаборатории. Это вообще беда многих долгостроев в физике.

(я не говорю что вебб хуйня - это самый охуенно чувствительный инструмент в космосе на сегодня, да и вообще единственный такой - просто надо понимать что могло получиться, если бы несколько ограничителей не существовали)
263 698705
А почему нельзя отправить миссию для обслуживания вебба?
264 698707
>>698705
см. >>697861
tl;dr: не на чем.

пилотируемая космонавтика так и застряла на земной орбите, все попытки улететь на сегодня кончились нихуём.

может быть возможен какой-то беспилотный аппарат для перезаправки не предназначенных для этого аппаратов через сопла (уже делали так), достаточно умный для того, чтобы задержка сигнала не была проблемой, но в любом случае такой нужно начинать делать уже сейчас, а никто не делает
265 698710
>>698707

>пилотируемая космонавтика так и застряла на земной орбите, все попытки улететь на сегодня кончились нихуём.



Очевидно что первый пункт за земной орбитой это Луна.
А раз люди туда не летают, значит человечество ещё не дозрело до этого.
266 698736
>>698681
Антисетипетух, спок.
268 698738
>>698682
Не читай жопой. Я говорю о Поверхности шара. Она - бесконечна. Ибо не имеет периметра ни в каком виде.
Бесконечный не равно бесконечнобольшой. Тебе этого в ПТУ не объяснят, добро пожаловать.
269 698740
>>698702
Мы за, но края не обнаружено.
Найдешь - покажешь.
270 698748
>>697821
То есть если планета будет состоять из идеального изотропного шара, допустим из железа, и покрыта одним океаном, то геоид будет эллипсоидом вращения? А в условиях Земли геоид отклонятся вверх под Гималаями и вниз в районе Марианской впадины, то есть в местах повышенной и пониженной плотности соответственно?
image.png1,1 Мб, 900x622
271 698750
Почему это шедевр не взлетел?
272 698751
>>698750
Шаттлостроение это провал. И совки кудахтающие про буран. Пусть радуются тому, что амеры первыми обосрались. Хоть денег столько не проебали на опыт провала.
273 698758
>>698750
Страна кончилась.
274 698764
>>698750
Ну вообще этот шедевр два раза таки взлетел. Потом больше не летал
275 698772
Объясните на пальцах в жопе, как телескоп может видеть "прошлое" Вселенной? Я правильно понимаю, что здесь такой же принцип как мы видим далёкие звёзды, но они далеко и их может уже не быть, а остался только свет? Так как тогда мы можем посмотреть раннюю Вселенную, реликтовое излучение? Нас так сильно откинуло взрывом, что свет не успел дойти до того места, где мы находимся? И куда вообще надо направлять телескоп, в место предполагаемого нахождения той самой сингулярности? А можно ли когда-нибудь будет заглянуть в планковскую эпоху?
276 698774
>>698772
Удваиваю
277 698785
>>698682
Вы заебали уже со своими научпокерами сравнивать Вселенную с воздушным шариком. Мне это нихуя не объясняет. Ибо шарик расширяется внутри моей комнаты. А внутри чего расширяется Вселенная? Это первое. И второе. Если ходить по шарику, то он бесконечный/замкнутый. Но можно (если ты мух например) полететь вверх от шарика и его покинуть. Если Вселенная ёбаный шарик, то как с неё полететь "вверх".
1d5e2e1664.jpg63 Кб, 1200x675
278 698787
Почему когда смотрю видосы с черными дырами, то прям жутко становится, помню как то скачал спейс энджин и полетел в черную дыру, чуть со страха тогда не обосрался. А один раз приснился сон, что землю засасывает в дыру, сразу же проснулся весь в поту
279 698812
Уэбб рассчитан на 5 лет, как я понял, из-за того, что ему придётся прожигать много топлива на поддержание орбиты. В чём проблема отправить туда через 5 лет топливный бак и дистанционно дозаправить?
280 698834
>>698772
Ты видеть можешь только то, что выпустило в твою сторону фотоны, это раз, и они до тебя дошли, это два.
Если нет мгновенного переноса сигнала от тебя до объекта наблюдения, то твои знания отстают на величину доставки сигнала.
Ты слышишь прошлое, видишь прошлое, но в обыденной жизни очень недавнее прошлое.
А в космосе можно увидеть седую старину.
281 698836
>>698785
А тут мы подходим к МЕРНОСТИ пространства.
Чтобы соскочить с шарика, тебе надо воспользоваться дополнительным измерением.
До тех пор ты просто бегаешь по кругу.

Расширяется она внутри пространства большей мерности, которого мы не видим, и даже не ощущаем, как время. Мы только догадываемся об этом по разным сторонним эффектам.

Гугли всякое про пространство Калаби-Яу, восхитись тассть.
282 698837
>>697359

> Красные карлики это совсем другое. Еще зона обитаемости


Вот даже взять обычную нашу земную жизнь. Бактерий.
Они спокойно смогут жить на темной стороне планеты вокруг карлика в приливном захвате, если температура нормальная и атмосфера хоть какая-то есть.
Была бы жидкая вода, да тепло.
А раз смогут бактерии, смогут и более сложные, это уже дело техники.
283 698841
>>698837
Не особо смогут, даже прогреваемая по всей поверхности планета имеет сложности с видовым разнообразием.
На американском континенте было гораздо меньше видов животных, поддающихся одомашниванию, и это сказалось на их способности конкурировать цивилизационно в том числе.
Так что в рамках одной планеты, если так прикинуть, нужно будет получить обитаемые пространства такие же по площади, как на вращающейся Земле, да еще в дальнейшем географически связанные, чтобы целые виды не уплывали на тектонических потоках в ебаный ад или ебаный холод.
284 698849
>>698785
Потому что ты еблан и не понимаешь, что математика это формальный язык, который не обязан описывать нечто что ты знаешь в повседневной жизни. Плоское двумерное расширяющееся пространство описывается метрикой diag(a(t)^2,a(t)^2), а топология у него может быть какая угодно, хоть тор, хоть сфера бывают плоские сферы, да
285 698851
>>698812
На 10 лет, так и хотят сделать
286 698860
>>698837
Ээээээээ. В твоей версии слишком много если. Да и в случае с красными карликами, там если вообще что-то живое и возможно, то какие-нибудь уберэкстремофилы. Тут надо подтягивать немножк других ребят для дачи пояснений, но если мягко говоря:

>А раз смогут бактерии, смогут и более сложные, это уже дело техники.


Это не совсем так работает. Чисто технически. Есть грубо говоря запредельный набор условий окружающей в среде. До тех пор, пока не будет доказано обратного. Ну ты и сам все знаешь.
>>698841
А вот и голос разума поразумнее подъехал. Это мы еще не наложили фактор стерелизации. Которого у нас (в сравнении) скорее не было, чем был.
287 698861
Помогите решить задачку?
Есть тело сферической формы радиусом 100 км и массой 1x10-20 кг. Каково будет значения ускорения свободного падения на его поверхности?
288 698864
>>698861
Бля. Я программист, физику не помню нихуя, но даже я легко могу подобрать формулы из памяти.

> массой 1x10-20 кг


В минус 20? Тогда мизерное.

{ F = ma
{ F = G x m x M / rr

a = G x M / rr
G = 6.67408 × 10-11 м3 кг-1 с-2 - гравитационная постоянная
M = 10-20 кг
r = 105 м
a = 6.67408 x 10-11 x 10-20 / 1010 м/сс = 6.67 x 10-31 м/сс
Если масса не в минус 20, а в плюс 20, то результат домножь на 1040
289 698865
Как контрить огромные расстояния в космосе? Судя по всему, путешествия со скоростью, близкой к скорости света, никогда не станут реальностью.
290 698866
>>698864
a = 6.67408 x 10-11 x 1020 / 1010 м/сс = 0,67 м/с2

Верно?
291 698867
>>698865
Пока никак.
Расклад на текущий момент, объяснит мистер Штерн.
https://youtu.be/o8FbMjH4pFM?list=PLvPeeS2DeP4eyQdt3-dqCw_XAyJBJdK9Q
292 698868
>>698866
Я в последней строчке объебался.
Да, у тебя верно.
293 698869
>>698867
>>698865
Поправ-очка. С роликом ошибся.
Вот тут.
https://youtu.be/wWD7YBgfuxo?list=PLvPeeS2DeP4eyQdt3-dqCw_XAyJBJdK9Q
294 698871
>>698865

> путешествия со скоростью, близкой к скорости света, никогда не станут реальностью.


Уже стали. Есть проект по разгону маленьких зондов до 1/3 что ли скорости света. Просто сейчас это нецелесообразно, и никак не применишь, но пулять зондами можно.
295 698874
>>698871
Зонды это неинтересно. Как люди полетят, чтобы не разъебаться от перегрузок?
296 698876
>>698874
Проблема вовсе не в перегрузках.
297 698881
>>698871

>Есть проект по разгону маленьких зондов до 1/3 что ли скорости света.


Это тот самый странный проект русского олигарха, который предлагает микрочипы массой в единицы граммов разгонять при помощи лазерного импульса с лазерной пушки на орбите Земли?
Он же накорман, нэ?
298 698884
>>698881
Так а что это разве невозможно сделать?
299 698894
Что общего у Вернера фон Брауна и Дмитрия Рогозина, кроме того, что в молодости были сторонниками национал-социализма?
300 698895
>>698894
Мужской пол.
Полный набор конечностей.
Деятельность связана с космосом.

Думаю, можно не продолжать.
301 698897
>>698860
Ну чего уж так жестить то. Будем себе воображать очень спокойного карлика, котрый стерилизациями не промышляет или бузит не часто.
Понадобится сверхземля, плотность поменьше, диаметр поболбше, на дальнем краю зоны обитаемости и глядишь где-то в тени рельефа что-то будет переживать даже периоды звездных вспышек.
302 698898
>>698895

>Деятельность связана с космосом.


проиграл
303 698899
>>698898
Да, тут подобосрался спизданув чутка, хотя формально технически на бумаге связана же.
304 698920
>>698772

>Объясните на пальцах в жопе, как телескоп может видеть "прошлое" Вселенной?


Вселенная имеет конечный возраст и свет идет с конечной скоростью. Соответственно, чем дальше в пространстве смотришь, тем более старые объекты видишь, потому что свету от этих объектов потребовалось больше времени, чтобы за время жизни Вселенной долететь до наблюдателя, чем свету от более близких тел.
305 698921
>>698785

>А внутри чего расширяется Вселенная?


Ничего, убери мысленно дополнительные измерения. Представь, что ты двумерный таракан, живущий в двумерной вселенной - на двумерной поверхности. Ты можешь ползать только в рамках этой поверхности, никакого другого измерения не существует. То же самое с нашей Вселенной, с той лишь поправкой, что она трехмерная, как и мы, живущие в ней.
306 698923
>>698865

>Как контрить огромные расстояния в космосе?


Никак, пилотируемая космонавтика дальше Луны и Марса вряд ли когда-либо куда-нибудь двинется. Так что про пилотируемые полеты к звездам можешь забыть. Исследовать Вселенную мы всегда будем лишь посредством всяких роботов и аппаратов, как сейчас, но никак не вживую.
307 698925
>>698897
Воображать можем много чего друг любезный, а по факту то...
Вот ты и я помрем раньше, чем эта идея найдет практическую реализацию(((( Раньше лет так на 500 (в лучшем случае). Может быть и на тысячу (раньше). Теоретически да: и туда залезть не помешает (вопрос аппаратуры еще очень серьезный).
308 698934
Мужики кто в Казани, на юго-западе Солнце заходит и на него можно в бинокль смотреть без филтров, оранжевый шар. большой. странно думтаь то все тепло - от него. А без него была бы -270 ледянуая пустыня. или - 273.10? -273.13?
309 698938
>>698934
Солнце оказывается заходит в Казани, интересный факт, я не знал. А встает где? В Бердичеве небось?
310 698947
С какой угловой скоростью/минуту уходит солнце за горизонт сейчас в Москве? размер диска - 30 минут, т.е. 1/360 части небесной полусферы? так мало??????

Как рассчитать длину дня в данное время на данной широте? Как(нелинейно из дугообразности?) меняется скорость движения диска солнца по небесной сфере в течение светового дня?
311 698950
>>698938
>>698934
Что ж вы творите содомиты
312 698953
>>698938
сегодня пасмурно и смог, я о том, что в него можно голым глахом смотреть
313 698956
>>698860
>>698841
>>698925
Вы чё, припизднутые, реально существующие земные бактерии могут жить литерали в любых условиях с жидкой водой и источником энергии.

Все нахуй, это единственные два условия. Ни толщина атмосферы, ни радиация, ни все остальное роли не играют.

Одомашнивание какое-то, чё несут, охуеть.
314 698961
>>698956

>с жидкой водой


>жидкой водой


Ну и кто тут припёзднутый?
1.png1019 Кб, 878x710
315 698976
Знаю что не по теме, но что за говно китайские братушки в Амур по своей реке Songhua Jiang заливают? И почему это говно не смешивается с водами с верховья Амура почти до самого Хабаровска? Почему наши экологи не бьют тревогу по этому поводу?
Ну и можно ли это как-то в космонавтике применить чтобы совсем оффтоп не был?
316 698988
>>698836

>Расширяется она внутри пространства большей мерности, которого мы не видим, и даже не ощущаем, как время. Мы только догадываемся об этом по разным сторонним эффектам.


Ясно.
317 698993
>>698976
Грязь, мелководье. Спрашивай лучше на sci/.
318 698995
>>698993
Заебал со своим обо/sci/, в прошлый раз когда я туда ходил, там было логово шизиков и битардов, которые нихуя не знают. Здесь тоже появились свои, правда.
>>698976

>Почему наши экологи не бьют тревогу по этому поводу?


>наши экологи


Кто? У нас оказывается есть экологи, и они ещё не сидят? Если так, то их даже Кайеркан-2020 не разбудил.
319 699003
>>698976
Как вариант, жидкости с настолько разными плотностями, температурами и составом, что требуется продолжительное время на смешивание.
320 699020
Что это за сияние на горзонте? В той стороне никаких заводов или жилых массивов нет, только лес.Да и от фонарей должно быть оранжевый покрас, а не белый. Регион южной Нижегоросдкой области.

Выходил на улицу смотреть — свечение очень яркое, будто днём.

Солнце? Тоже навряд ли. В стеллариуме район наблюдений выглядит примерно так.
321 699038
>>698740
Как ты его обнаружишь, то если ты дал ше видимой вселенной заглянуть не можешь?
вселенной шара тоже не обнаружено. Реликтовое излучение показывает, что вселенная flat.
Так что за край 1
За шар (впуклую кривизну) 0
322 699039
>>698785
Двачую. Заебали.
Вселенная это диск.
323 699041
>>698787
Потому что неизвестность пугает.
А тело человека такое слабое по сравнение с гигантскими силами черной дыры. Вот и страшно.
324 699042
>>698849
хуйня твоя маиематика, написали какие то символы на уровне завета божьево, а пруфов 0.
325 699046
>>699038

>вселенная flat


Опровергается полпрыгиванием.
326 699048
>>698947
Я ебу что ли?
327 699049
>>699046
дибил блядь
даже обьяснять тебе лень, иди нахуй тупое животное
328 699056
>>698956

>Ни толщина атмосферы, ни радиация, ни все остальное роли не играют.


Ага, как скажешь.
1640744503245.jpg73 Кб, 700x936
329 699058
>>699038

>вселенная flat


Ну или она настолько большая, что мы не в состоянии увидеть кривизну
image.png1,8 Мб, 1920x1080
331 699068
Зачем инженеры при строительстве телескопа Уэбб работают в таких костюмах и перчатках? Аналогичное видел и с Персеверанс.
332 699070
>>699068
Такое даже не на самых наукоемких производствах используют. Двигатели тойты в японию в такой же защите собирают. Стерильность, епта
333 699072
>>699070
А зачем она нужна? Что изменится, если ты пальчиком туда своих бактерий намажешь?
334 699073
>>699068
У вебба очень точное зеркало, не дай бог заляпаешь.
У персерверврбртьфу другие соображения - не занести своих микробов за Марс, чтобы их потом приняли за внеземную жизнь, или чтобы они случайно и не стали ей.
335 699074
>>699049
И зачем же ты такое тупое животное?
iStock-522322030d850d850.jpg141 Кб, 1000x667
336 699079
Каким образом управляют спутниками и всякими марсоходами? Пока сигнал до них дойдет, то пройдет же нихуевое время, там же наверно пиздец какая задержка и наверняка сама скорость передачи данных очень маленькая. Они типо посылают сигнал, а тот же марсоход сделает заданное действие только через несколько часов или дней?
337 699083
>>699079

>Каким образом управляют спутниками и всякими марсоходами?


С помощью сигналов, которые распространяются со световой скоростью.
338 699084
>>699083
Таких не бывает
339 699089
>>698976
На китайской части бассейна амура проживает где-то 50 000 000 китайцев, с российской наверно и десятка не наберется
так что

>но что за говно


Сам-то как думаешь?
340 699090
>>699079

>Они типо посылают сигнал, а тот же марсоход сделает заданное действие только через несколько часов или дней?


Да. У марсоходов есть небольшая (очень небольшая) степень автономии, плюс им по картинке с камер задают траекторию вручную если нужно.

>спутниками


Если речь об ИСЗ, то там не особо большая задержка, но и телеуправления им не нужно, за небольшими исключениями (например оператор с земли управляет роботами на МКС вполне себе в реальном времени).
А АМС в дальнем космосе в основном выполняют заложенную программу полёта - операторы лишь принимают решение выполнять или нет тот или иной шаг, либо меняют эту программу на ходу.
341 699093
>>698923
Что значит вживую? Человеческий мозг это вполне объективная вещь, и воссоздание его функций лишь вопрос времени.
Сознание вне мешка с мясом точно так же может пилотировать корабль, и точно так же такая космонафтика будет называться пилотируемой.
342 699094
>>699084
Утверждаешь - доказывай.
343 699096

> А могут быть какие-то иные существа? Как в интерстелларе?


> Нет, в пространствах большей размерности сложные структуры невозможны в принципе, не смогут сформироваться.


Почему бы не допустить что им и не нужно формироваться или у них какое-то совсем иной способ существования?
344 699104
>>699093

>Что значит вживую?


Значит, что ты не сможешь сесть за штурвал звездолета и никуда не полетишь.

>Сознание вне мешка с мясом точно так же может пилотировать корабль, и точно так же такая космонафтика будет называться пилотируемой.


Пиздеж, сказочки, маняфантазии.
345 699105
>>699096

>Почему бы не допустить что им и не нужно формироваться


Тогда их не существует, лол.

>или у них какое-то совсем иной способ существования?


Да, как я и сказал, их способ существования - несуществование.
346 699106
>>699104

>Пиздеж, сказочки, маняфантазии.


И что в мозге есть такого, что нельзя было бы воссоздать? Святой дух?
347 699117
>>697053

>Потому что это газовый шар


>газовый


поссал тебе на литсо
348 699146
>>699104
В чем проблема изобрести Варп-двигатель в будущем?
349 699154
>>699146
Невозможность его существования
350 699155
>>699146
Нет проблемы, но для этого его нужно заранее разогнать до скорости света. Идея этого двигателя может создавать пузырь, который теоретически может двигаться быстрее света, но практически в нашем мире она не разгоняет пузырь до скорости света. Как мы придуем ускорение в космосе, выбрасывая вещество, можно горяжее, это придует кораблю испульс. Скорость корабля ограничена макс скоросью выбрасываемой материи из сопла или взаимодействием полей похуй.
И дело не только в том, что у нас нет таких техноинженерных умений, хотя и в этом тоже, но и в том, что это впринципе не даст тебе хуярить быстрее света, даже если все получиться как в модели.
Сверх этого требования там такие, что нужно на небольшом участке поместить огромное количество энергии с положительной и отрицательной массой. Надо как то взять юпитер и переварить в чистую энергию и поместить в одной точке и рядом столько же отрицательной отрицательную. Как это вообще возможно, чем ты будешь ее держать то бляд , наноматериалом или может левитацией? Нереальная хуйня
Даже если на это похуй, то все равно всесхлопнется в черную дыру еще до достижения нужных энергий.

Варп - это какой матеметический высер фаната старворс, который ничем другим как фанатским фантазерским высером оформьенным в какую то матоно-модельку,, не учитывающую почти нихуя и экстраполирует уравнеиние Энштейна в абсурд, хотя уравнение Энштейна даже даже скорость звезд в галактике неверно описывает.

Щизики блядь.
351 699156
>>699155
Ну да, но в 19 веке полеты в космос тоже были всего лишь научной фантастикой.
352 699157
>>699156
ПТИЦЫ ЭТО ДЕЛАЛИ.
Птицы нахуй.
Тупые ебучие биороботы

То есть полет был изобретен эволюцией отбора.
Но сейчас вроде вокруг нет никаких намеков на том, что эта идея не более чем сектанство фантазеров. Нельзя же брать просто любую фантазию, которая даже в теории не описывает как превысить скорость света, она лишь говорит что это не запрещено, но не значит что возможно это осуществить.
Это триллиона математический фантазий о нашем мире. Варп это техно фэнтези с фанатом математиком.
353 699158
>>699156
полеты птиц
проебался анон.

Но даже с полетами в космос это просто бочка с горящим пожаром внизу, которую нужры было оптимировать, что подлетеь над землей.
Это было для человечества последние 2000 лет уже hard sci фаем, ибо это хотя бы гипотетически можно было представить осуществимым.
Совсем уже ясно стало с открытия атмосферы земли.
354 699163
>>699056
Но это так. Минимальная толщина атмосферы и температура задаются условием по жидкой воде.
Причем ей даже не обязательно всегда быть жидкой, можно периодически.

На радиацию строго похуй, бактерии даже в активной зоне ядерного реактора могут выживать, чё им ваши выпуки карликов, даже не заметят.

Просто на Земле мягкие радиоактивные условия, вот большинство организмов к ним и не приспособлены.
На радиоактивной планете изначально выживать будут приспособленные, а остальные просто сдохнут.

Как сдохли на Земле все те виды, что не вписались в рыночек.
355 699191
>>699106
Когда воссоздашь 86 млрд нейронов с триллионами нейронных связей, тогда и возвращайся, а сейчас ты просто в лужу пукаешь, вероблядь.
>>699117
Почему ты порвался, мелкобуквенный антисетипетушок?
356 699193
>>699146
В том, что варп-двигатель - не более чем научно-фантастическая поебота. Или даже не научно-фантастическая, а откровенно фэнтезийная хуита.
Безымянный.png606 Кб, 1501x1123
357 699202
>>699155

>Сверх этого требования там такие, что нужно на небольшом участке поместить огромное количество энергии с положительной и отрицательной массой. Надо как то взять юпитер и переварить в чистую энергию и поместить в одной точке и рядом столько же отрицательной отрицательную.


По последним подсчет нужно намного меньше отрицательной массы
358 699213
>>699020
Дополнительное фото вчерашнего свечения, а также изображение того, как это выглядит сейчас (никак).
359 699242
>>697264

А что там интересно?
360 699263
>>699105

> Тогда их не существует, лол.


Это в привычном нам смысле, потому что их существование находится вне пространства-времени. Может быть им не нужно было формироваться, они там всегда были, либо сформировались что потом ушли туда
361 699276
>>699163

>даже в активной зоне ядерного реактора могут выживать


))))))))))))))))))))))))))))))))))))))))))))))))))))))))))))))))))))))))))))))

>даже


>выпуки карликов


Карлик, кого ты пытаешься наебать. Какой рыночек нахуй, ты вообще что с чем сравниваешь? Вопрос риторический.
image.png313 Кб, 512x351
362 699291
>>697045 (OP)
Я правильно понимаю что любой косяк у термоядерного реактора значит лишь затухание реакции внутри и никаких пиздецов? Радиаций не выбросится, взрывов не будет?
Почему их не строят массово?
363 699292
>>699020
>>699213
Викимапию открой своей местности, там может что-то эдакое у тебя построили где ебический прожектор херачит?
image.png5,6 Мб, 2863x1425
364 699293
>>699072

>Что изменится, если ты пальчиком туда своих бактерий намажешь?


Картинка хуже будет.
365 699295
>>699156

>Ну да, но в 19 веке полеты в космос тоже были всего лишь научной фантастикой.


А еще в 19 веке ходьба по воде была научной фантастикой и делом отдаленного будущего, только это... Ты не можешь ходить по воде привязав шарики к себе. Объективная реальность бытия проводит по губам своим шершавым.
Это я к чему. Не всё о чём фантазировали тогда сбылось, это совершенно не аргумент, они фантазировали о чём угодно, но ты помнишь только то что сбылось вроде космических полётов.
366 699296
>>699202
А она вообще существует? Это же чистейше математическая концепция "если мы тут минус поставим то полетим-похуячим", где эта отрицательная масса ирл?
367 699301
Подвидем итоги про варп двигатель:

Если сконцентрировать столько +/- энергии в малом обьеме ты получишь белую и черную дыру.

Манипуляции такими энергиями и их правильное расположение вокруг корабля - даже гипотетически это себе невозможно представить. Как это сделать инженерно?

Все равно варп пузырь не разгоняется до скорости света сам, его надо как то разогнать. А как? Как разгнать обьект выше скорости света, если разгонять можно только веществом, которое не может двигаться быстрее скорости света? Тебе известны хотя бы гипотетически другие формы разгона? Для разгона потребуются тахионы лол, которые не существуют ибо они нарушают принцип причинрости.

Отрицательной массы не существует.

Радиация хоккинга на границе пузыря и прочие неучтенные глюки и эффекты - неясно можно ли вообще выжить внутри этого пузыря.

Куда отводить тепло из пузыря и как?

Короче с таким же успехом мы можем обсуждать полет к альфацентавре на драконе накачанным магическим зельем.
368 699303
>>699301

>Подвидем


Дальше не читал. Нет смысла ожидать чего-то осознанного после такой нелепой дурости.
369 699307
>>699263
У тебя шизофазия, завязывай с боярой.
370 699309
>>699291

>Я правильно понимаю что любой косяк у термоядерного реактора значит лишь затухание реакции внутри и никаких пиздецов?


Да.

>Почему их не строят массово?


Пытаются строить побольше сейчас, но это очень дорого. Если таки люди смогут по-настоящему освоить термоядерный синтез, то за ним безусловное энергетическое будущее человечества.
371 699311
>>699296
Ну сегодня не существует, а через 100лет создадут
372 699313
>>699293
>>699073
Так я не про зеркало говорил, здесь вопросов нет. Там вообще все рабочие и инженеры в костюмах. А на картинке же в Хаббле чел накосячил по-другому, линзу что ли не так закрепил.
373 699314
>>699291
Нет, неправильно. Любой термоядерный реактор это радиационно-опасный объект. Вполне возможна авария в водяном контуре (который может быть и под давлением, как в ядерных реакторах). Контур разрушается ->пар разламывает ректор -> кипящая вода размывает активированные конструкции, разливается наружу и создаёт загрязнение. Всё, это радиационная авария. А активированные конструкции будут в ЛЮБОМ термоядерном реакторе, потому что они все излучают нейтроны. И протон-бор тоже излучает нейтроны. Единственная реакция ever которая не излучает нейтроны, это гелий3-гелий3, но такой реактор очень сложно сделать

>не строят массово


Так не могут потому шьто алё?! Реактор выделяет тепло, потом обычная тепловая машина его преобразует в электричество. КПД тепловой машины 35%, электричества значит будет в 3 раза меньше чем тепла. А сам реактор как раз и потребляет это же электричество. Поэтому чтобы он просто запыхтел сам на себя, нужно энерговыделение в 300% от электропотребления реактора. А сейчас, в итт некст году, могут только 100%. Расти ещё в 3 раза, и даже не в 3, а в 5 или даже 10. Потому что циркуляция энергии в реакторе раздувает его обвязку, если он выдаёт 1 ГВт электричества, а внутри крутится вдвое больше, то все эти электромеханизмы будут оче большими и оче дорогими. Нормальный реактор должен выделять в 10 раз больше от потребления!
16398552030970.jpg69 Кб, 600x800
374 699317
>>699307

> У тебя шизофазия, завязывай с боярой.

375 699318
>>699291
Термоядерный реактор хуярит высокоэнергетичными нейтронами, а они заставляют фонить всё, во что ударяются. Существуют анейтронные реакции и реакции со сравнительно малым выходом нейтронов (не только гелий-3), но их зажигать на порядок сложнее - сейчас бы хоть обычные научиться с положительным выходом.

Общая энергия и активность в беккерелях, запасённые в термоядерном реакторе, действительно намного меньше таковых в ядерном.
- ядерный реактор работает непосредственно на всём своём многолетнем запасе топлива сразу и одновременно, и выжигает лишь небольшой его процент, а при бабахе огребаешь проблем со всем топливом
- термоядерный содержит только то, что в данный момент участвует в контуре. Т.е. если его разорвёт, будет максимум маленькое локальное заражение без сильных выбросов, и то под вопросом. А не Чернобыль или что похуже.

>Почему их не строят массово?


Потому что не умеют.
376 699321
>>699318

>анейтронные реакции


>не только гелий-3


нет друг, true-анейтронная реакция это ОНЛИ гелий-3. все другие-остальные топлива кроме основной реакции осуществляют и побочные, и вот в побочных всегда выделяются нейтроны. В протон-боре например энерговыделение в нейтронах около 0,1%, в протон-азоте и протон-кислороде и того выше, даже протон-литий выделяет нейтроны. конечно загрязнение при аварии будет небольшиим, но это всё ещё радиационно-опасный объект, тебе нужна лицензия, и тебе нужна плошадка за городом, тебе нужно утилизировать радиоактивные отходы (активированные конструкции)
только лишь швятой гелий3 позволит создавать распределённые сети и модульные реакторы прямо в городских кварталах, и никаких отходов!
377 699334
>>699321

>и реакции со сравнительно малым выходом нейтронов


Вот, ты уронил.
378 699409
>>699291

> Я правильно понимаю что любой косяк у термоядерного реактора значит лишь затухание реакции внутри и никаких пиздецов? Радиаций не выбросится, взрывов не будет?


Ну в целом, да. Может ебануть какой-нибудь холодильник или пукнуть плазмой туда, куда не следует, но это скорее означает, что ремонтировать ещё 20 лет надо будет.

Радиации там мало.
379 699417
>>699301
Потому что это все отсталые представления, использующие старую физику. Надо искать возможности в новых теориях, пробовать дергать струны или поля, отвечающие за пространство и время, а не пытаться нагородить материи и энергии, которые всего-лишь являются проявлением струн полей в нашей четырехмерной свертке. Получится ли это сделать не залезая на планковские масштабы - непонятно, но копать надо туда по-любому.
380 699418
Объясните что такое сингулярность на примере говна и палки
381 699420

>Благодаря удачному стечению обстоятельств во время корректировок курса телескопа удалось сэкономить значительное количество топлива. В NASA считают, что теперь у JWST достаточно энергии, чтобы оставаться в эксплуатации еще десять лет сверх запланированного ранее срока.


Собственно, вопрос, а че нельзя сразу делать дополнительный бак и переодически заливать туда топливо?
Ну или отправлять космозаправщика, который будет менять топливные элименты? Какие бы они там не были.
Тогда бы телескопы были бы почти вечные.
382 699425
>>699418
Специальная область в центре черной дыры, в которой ломаются все существующие физические теории и теряется смысл самого понятия пространства. В самом начале Вселенной было аналогичное состояние.
383 699429
>>699418
К говну и палке добавляешь ещё дохулион говна. Всё это слипается в грандиозную кучу фекалий. Продолжаешь добавлять ещё говно, под давлением собственной гравитации внутри кучи зажигается термоядерная реакция. Продолжаешь срать, и в конце концов самогравитация сжимает твоё говно меньше радиуса шварцнеггера, и всё это непотребство резко схлопывается в сингулярность. Потом тебе отрывает ноги и распидорашивает об горизонт событий
384 699430
>>699420
Космозаправщик для аппаратов, для этого не предназначенных, уже тестировали на низкой орбите год или два назад. Может статься что и к веббу отправят лет через 10, чтоб долить горючки.
385 699438
>>699430
Это ДРУГОЕ. Низкая околоземная на порядок легче в маневрах и телеметрии. А точка Лагранжа и так очень не устойчивая, где любой пук распидорасит орбиту. И для стыковки придется потратить больше топливо, чем останется для дозаправки.
386 699452
>>699438
Каких манёврах? С баллистической т.з. вообще абсолютно нихуя разницы нет, в лагранже наоборот намного меньше скорости. Телеметрия вообще не понял причём тут. Если ты про автономность заправщика - вся суть эксперимента была в автономной стыковке без участия оператора и аппарата-приёмника, если не считать участием то что он был стабилизирован и не вращался.

>для стыковки придется потратить больше топливо, чем останется для дозаправки


Нихуя, полная хуйня, лагранж очень низкоэнергетичен - выйти на лассо или лиссажу или что там у вебба, а дальше чтобы подрулить нужны мелкие пуки.
387 699488
>>699418
в там сидит говно по имени тимоти и палкой сбивает всё что пролетает рядом, даже фотоны, даже нейтрины, даже сраные базоны сраного хигса
388 699514
Откуда люди, занимающиеся SETI, знают, что сигнал от внеземной цивилизации не ослабнет проходя через межзвездную среду до состояния невозможности его детектировать? Ведь очевидно, что никто передатчик не помещал около Проксимы Центавра и прохождение сигнала от него до Земли не тестировал.
389 699518
>>699514

>Откуда люди, занимающиеся SETI, знают, что сигнал от внеземной цивилизации не ослабнет проходя через межзвездную среду до состояния невозможности его детектировать?


Ищут сигнал там, где можно найти и принять.
390 699519
>>699518
Ну это не ответ. Вот направили они на звезду эпсилон эридана тарелку, и что? А если по пути все сигналы затухают, а они этого и не знают? Может там и есть передающая сигнал цивилизация, но ни она ни мы не знаем, что сигнал не проходит между звездами, сильно тухнет
391 699520
>>699519
И из этого делается ошибочный вывод, что мы одни, парадокс ферми
AA16909E-185C-4F1D-BA10-9AFD29C9AF5F.jpeg366 Кб, 2208x451
392 699521
Гуру поздравил Егора
393 699523
>>699521
Почему на этой доске и в этом треде? Ты же это в /po/ хотел запостить, но был бухой и ошибся.
394 699524
>>699523
Бля, точно, в хрю хотел
395 699560
>>697045 (OP)
Почему Новый год наступает не в момент солнечного солнцестояния, который в прошлом году выпал на 21 декабря в 15.59, а просто в 00:00 рандомного дня?
396 699571
>>699560
Когда выбирали летоисчисление, астрономией руководствовались постольку-поскольку.
Одно время было 10 месяцев, март первый был.
Одно время была ротация между 12 и 13 месяцами чтобы в общем и целом за много лет не сдвигались даты.
В общей картине это не имеет никакого значения, это не дата которая привязана к астрономическому явлению, это такая же выбранная дата как 8 марта или день флота.
ыгт-ир.png142 Кб, 1863x1111
397 699573
с новым годом космачи!!
398 699577
>>699519

>А если по пути все сигналы затухают


Им не от чего затухать, физика радиоизлучения известна. На ровном месте все просто так не может взять и затухнуть в ноль, не маняфантазируй.
399 699578
>>699560
Потому что так удобно людям, очевидно.
400 699581
>>697126
ВСЕ СОЗДАЛ БОХ
@
Я ЕДИНСТВЕННЫЙ, КТО С НИМ МОЖЕТ ПОРЕШАТЬ
@
ДАВАЙ БАБЛО ЗА ОТПУЩЕНИЕ ГРЕХОВ
@
ВОТ ПРАЙС
@
АЙФОНЫ? ЧТО ЭТО ВАЩЕ?
@
СЖЕЧЬ ЕРЕТИКА!
401 699585
>>699581
такто бох - лошара и мудак, нормальный бох сделал бы соски на ягодицах, штоб их пичкать во ввремя ебли
402 699633
>>699581
Зачем ты противопоставляешь церковь и науку, когда много веков церковь была покровителем науки, а монахи занимались научной деятельностью? Зачем ты плюешь на чувства известных учёных, многие из которых были глубоко религиозными людьми?

Ты что, типичный подписчик аметистовых пабликов с цитатами в ВК?
403 699646
>>699633
Астрология тоже "наука", мой друх.
И чему там покровительствовала церковь - вопроооос. Больше всего она покровительствовала сбору бабла с баранов. Зачем? Ну так...
Какие технологии взлетели под эгидой церкви? Никаких.
Только ряд практик по цензуре, монополизации инфопространства, репрессиям в адрес любых явлений, нарушающих статус кво.

Веровали ученые в Бога, а не в церковь. С ней они срались.
404 699648
>>699646

>Астрология тоже "наука"


Главное свойство науки - предсказание событий на основе моделей
Что может предсказать астрология? Что у водолея на этой неделе будет счастье, а рыбы потерпят неудачу?
Хех
мимо
405 699649
>>699646

>И чему там покровительствовала церковь - вопроооос


И в чём вопрос?
406 699650
Возможно ли обеспечить человеку комфортный подъём на орбит без высоких перегрузок? И всегда ли это будет долго?
407 699651
>>699425

>теряется смысл самого понятия пространства


А какое именно понятие пространства в релятивистской механике и как он теряет смысл?
408 699655
>>699648
Что предсказывает наука лингвистика?
Ничего.
image.png162 Кб, 636x440
409 699656
>>699655
Так половину гуманитарок можно выкинуть...
410 699663
>>699646
Никакого вопроса здесь не стоит. Только благодаря церкви мы получили ренессанс. Только в пределах стен церкви в средние века существовала интеллектуальная мысль и только священнослужители занимались наукой в то время. И жажда денег отдельных личностей или отсутсвие технологий не опровергают эти слова.
411 699667
Почему бы вам не съебать нахуй со своим говном из треда тупых вопросов?
412 699685
>>699656
Именно эти "науки" и оберегала церковь, как ни парадоксально.
413 699686
>>699655
Что предсказывает наука математика?
Ничего.
414 699687
>>699651
Невозможно померить никаких расстояний и длин, не работают никакие "линейки".
Стикер255 Кб, 500x500
415 699688
>>699686
Результаты математических операций.
416 699689
>>699633
В /re пиздуй, вероблядь, и там обсуждай своих воображаемых бородатых мужиков на облаке.
417 699690
>>699686

>наука математика?


Математика - это не наука, лол блядь. Это язык науки. Чистая математика не имеет никакого отношения к реальному миру.
418 699693
>>699690
Выписана из наук, понял вас.
419 699694
>>699690
Вроде кто-то сказал, что математика — царица наук
420 699696
>>699663
Ренессанс мы получили благодаря баблу от торговли специями. Как и великие географические открытия.
Абсолютно неважно, были бы у них какие-то сорцы античности или нет. Не этим определялся движ.
Священнослужители занимались религиозными ритуалами. Церковь как централизованная структура запретила хирургию, и многое другое, чем обычно занимались монахи.
Абсолютно нулевой прогресс тому свидетельством.
421 699697
>>699693
Она и никогда не была в их числе.
>>699694
Мы говорим о том, что по факту, а не о всяких ебанутых абстрактных, ничего не значащих высказываниях типа "феласафея царица наук" или "мотематека царица наук". Каждый понимает под этой хуйней, что хочет. И с той точки зрения, что современная математика является наиболее адекватным языком описания реального мира, что она дает основу для естественных наук, можно ее назвать "царицей наук", но это опять-таки абстрактная хуита без конкретики. Так что лучше уточнять, чем конкретно является математика в контексте современной науки.
422 699698
>>699686

>Что предсказывает наука математика?


Вся наука математика это сплошное предсказательство.
Ты тупой.
image.png64 Кб, 239x211
423 699699
>>699698
Подожди, так анон выше пишет что это не наука.
Вы разберитесь уже между собой, а то нихуя не понятно, наука это или нет.
424 699700
>>699699
Зачем?
В целом-то тест простой - церковники не занимались математикой. Значит это наука.
425 699701
>>699700

>церковники не занимались математикой


Декарт, Бэйкон, Галилей, Паскаль
426 699702
>>699701

>Декарт


АХАХАХАХАХАХАХА
427 699703
>>699701
хахаха блять, математики прикрывались церквухой просто, чё ты как маленький наивный карлсон?
image.png293 Кб, 640x360
428 699704
>>699702
>>699703
Сливаешься, так с достоинством попробуй это делать. Хотя что это я, двач же, какое достоинство.
429 699706
>>699703
Да, особенно Епископ Декарт и Кардинал Паскаль.
Первый блядь по войнам кочевал дохуя времени, натурально дырявя людей куском арматуры, второй был чахлым сынком налогового чиновника, которого родители от себя далеко не отпускали, предполагая, что деточка скоро сама и так помрет.

Что там навычисляли тыщи кардиналов-епископов за сотни лет? Ничего. Один Декарт срет им всем по полной шляпе за один присест.
изображение.png383 Кб, 1042x496
430 699718
Спрошу глупый вопрос. Почему орбиты всех планет нашей системы лежат почти на ровном диске? Почему одни планеты не летают условно говоря сверху-вниз, другие так как сейчас. И почему не запускают аппараты условно говоря вверх? Там нечего исследовать?
431 699719
>>699718
Потому что из буквального диска планеты и образовались.
Диск потому что газопылевое облако вращалось, вращаться в двух плоскостях нельзя, все к одной и срослось
432 699727
>>699696
Одно бабло не решает, завали ты хоть всю Европу богатством, если нет развитой и распространённой интеллектуальной мысли, никакое бабло не поможет. Все выдающиеся умы средних веков - служители церкви и само собой у них была своя аксиология - Бог и все возможные доказательства его, а так же упрочнение авторитета церкви, а потом уже изучение боговой природы. А всякие технологии сто лет им не нужны были вообще. Каролингское возрождение, возрождение 12 века - всё дело рук священников. Сеть школ и университетов в Западной Европе. Монастыри, аббатства, где тысячи монахов старательно переписывали и сохраняли греко-римское наследие. Тут долго можно перечислять. Они создали массивный "субстрат", на котором начали появляться декарты, ньютоны, канты и прочие великие умы. Этого не могло быть раньше просто исторически, когда в 476 году Европу заселили толпы германцев, которые вчера жили племенным строем и приняли принципиально новое христианское мировоззрение взамен старому мифологическому. Я мог бы в подробностях расписать весь этот тысячелетний процесс, но мы находимся на спейсаче и тут это вообще оффтоп.
433 699731
>>699577
В космосе много пыли, газа и камней. О них затухание идёт.
434 699732
>>699718

>Почему орбиты всех планет нашей системы лежат почти на ровном диске?


Потому что все сформировались в одном протопланетном диске.

>почему не запускают аппараты условно говоря вверх?


Запускают их в любом направлении.
435 699733
>>699731
Это все учитывается при поиске сигналов от других цивилизаций. Говорю, физика радиоизлучения людям известна, ты не открыл Америку, до тебя уже все учли.
436 699734
>>699727
Репорт веробляди за ебаный флейм.
437 699735
>>699734
Так это ты тут рвёшься от неприятных фактов. Реальность она такова, порой может травмировать.
438 699741
>>699733
Я не претендовал на открытие чего-то нового. Но ведь действительно радиосигнал довольно легко теряется, разве не так? То есть мы вот на Земле фоним сидим во все стороны, а он довольно быстро сходит на нет, и получается довольно небольшая сфера по меркам той же галактики. Так же и мы не можем толком ничего поймать здесь. Если я не прав, просвети меняю
439 699746
>>699727

> если нет развитой и распространённой интеллектуальной мысли


Она появляется, поскольку образовавшийся досуг надо куда-то девать. Ренессанс это про купцов и их республики. У них образовывалось время и бабло от сверхдоходной торговли, причем это было надолго, на сотни лет, чтобы все сильно изуродованные войнами пращуры вымерли, и поколения потомков не знали тягот, крутили колеса семейного бизнеса и спасались от рутины в интеллектуальных упражнениях. Это создавало спрос, и жырным барчукам начали продавать всякие древние книги за многаденяк, обучение языкам, на которых были написаны те книги, и тому подобное.
У религоблядей это вызывало жжение пониже спины ибо разрушало их монополию на информацию.
Купцы же бывали много где, в том числе там, где нет церковников, а книги - есть.
440 699749
>>699735
Вероблядь, тушись.
>>699741

>Так же и мы не можем толком ничего поймать здесь.


Можем, если это мощный сигнал. Такие сигналы как раз и ищут. От радиопульсаров и магнитаров ловят радиосигнал легко, а они могут находиться в десятках килопарсек от нас я уже не говорю о пульсарах и магнитарах из других галактик, а также о квазарах, сигналы от которых ловят так же без всякого напряга. Подобные сигналы может генерировать и какая-нибудь очень развитая цивилизация. Ну и более слабые сигналы тоже ищут, вдруг ближайшая ВЦ сидит недалеко от нас это очень маловероятно. От нас, если память не изменяет, при современном уровне развития технологий радиоизлучение распространяется на несколько десятков парсек вокруг. Аналогично и мы могли бы принять сигнал от ВЦ, которая находится от нас примерно на таком же расстоянии. Короче, особых проблем с затуханием сигнала нет. Поймать сигнал реально, и их постоянно ловят. Правда, большая их часть находит естественное объяснение, не связанное с деятельностью ВЦ, но есть и сигналы, которые пока таких объяснений не имеют скорее всего, они появятся в будущем.
441 699754
>>699749
Зачем высокоразвившись куда-то сигналить?
Это из разряда предположений, что города будущего утонут в конском навозе.
Скорее наоборот, перестанут быть заметны. Зато многое увидят.
442 699756
>>699746
Я не спорю. Купцы, время и бабло у богатеньких итальянцев - это тоже условие для развития науки, но оно не было единственным. К тому времени уже образовалась обширная сеть школ, университетов и библиотек. Пример, конечно, не удачный, но если ты в тот же Таджикистан закинешь много денег, там не будет никакого развития, если нет университетов, преподавателей. Опять же повторю, что церковь стремилась в первую очередь обосновать бытие бога и прочие штучки, а обширная сеть универов оказалась побочным результатом сего дела. Я сам антиклерикал, сторонник секуляризма и агностик, но я привык признавать факты, поэтому мне не больно признавать, что католическая церковь дала буст науке, в отличии от рвущегося анончика, который отписался ниже тебя.
443 699763
>>699754

>Зачем высокоразвившись куда-то сигналить?


Радиоизлучение много какая техника испускает. С чего бы технике гипотетической высокоразвитой ВЦ не испускать мощное радиоизлучение? К тому же, они могут намеренно посылать именно мощные радиосигналы, понимая, что радиодиапазон - наилучший диапазон электромагнитного спектра для связи с другими цивилизациями, потому что все более-менее развитые цивилизации (как наша или чуть более продвинутые) будут оставлять следы и искать следы других цивилизаций именно в радиодиапазоне.
444 699767
>>699756
Как раз таки было.
На таких же основаниях науки саморазвивались в античности. Тоже купеческие полисы, торговля по всей ойкумене, распространение сложных технологий и привозных материалов.
Как только все вернулось к такому же товарообороту - пожалуйста, науки, и прочие культурные явления.
Голодающие монахи могли максимум отрабатывать заказы знати на переписывании ветшающих книг, и держать предприятие по продаже и производству этих книг - библиотеку.
445 699768
>>699763

>С чего бы технике гипотетической высокоразвитой ВЦ не испускать мощное радиоизлучение?


С чего бы лошадям будущего не срать на мостовую в 20 раз интенсивнее?
446 699781
Мне приснился сон, в котором на Землю упал метеорит, что заставило задуматься о том, что было бы, если бы это реально произошло.
Размер оценить сложно, но метеорит заслонял примерно 1/6 неба в момент приближения (он летел намного западнее, описывая дугу вокруг планеты). В момент его падения происходило следующее: на секунду-две ослабилась гравитация, люди слегка поднялись в воздух (во сне это было объяснено тем, что у метеорита была большая масса). Был слышен то ли свист, то ли гул, и резко заболела голова (из-за перепадов давления). Когда метеорит упал, был слышен гул и жужжание, а планету начало как бы немного кренить в сторону, из-за чего люди съезжали, как на раздвижном мосту (во сне по новостям сказали, что это из-за того, что Земля на некоторое время ускорила вращение и что часы надо перевести на несколько часов вперёд). Потом всё улеглось, и в новостях было описано, что в целом пострадавших мало, нам повезло и метеорит упал где-то в Тихом океане, утопив много островов и небольшую часть суши некоторых стран.
447 699783
>>699781

> на секунду-две ослабилась гравитация, люди слегка поднялись в воздух


Что за нахуй, мне то же самое приснилось как-то раз, но это было когда сраная земляшка упала в юпитер. Я задохнулся падая в юпитер.
448 699790
>>699768
Аналогия - не аргумент.
449 699796
>>699790
Тогда ты слился.

С чего ты взял, что электромагнитное поле прямо такая безальтернативность? Его тоже когда-то открыли и учились им пользоваться.
А до того только звуковые волны есть наилучший способ передавать информацию. И если где-то есть цивилизация, то она должна как-то там пердеть, кряхтеть и шуршать.
Давайте же искать цивилизации на слух!

Аргументы достойные блохастой псины.
450 699809
>>699763
>>699796

>Радиоизлучение много какая техника испускает.


Можно получше выдумать аргументы. Ну хотя бы повсеместное использование методов расширения спектра. Уже сейчас много где ставят тактовые генераторы с расширением спектра для уменьшения помех, а если они будут везде? Толку то с такого радиоизлучения.
Или вон Юпитер тоже "испускает", а сколько экзоюпитеров нашли по радиоизлучению?

>>699749

>От нас, если память не изменяет, при современном уровне развития технологий радиоизлучение распространяется на несколько десятков парсек вокруг.


Это все глупости. Зависит от размеров приемной антенны. Точно так же свет от городов распространяется на десятки парсек. Если ты хочешь заявить, что такое радиоизлучение и с такого расстояния можно было бы принять на существующий земной радиотелескоп, то без ссылки такие утверждения лучше не давать.
451 699811
>>699809

>Или вон Юпитер тоже "испускает", а сколько экзоюпитеров нашли по радиоизлучению?


Вот-вот.
Такой же тупняк, как искать мегаполисы по облаку смога. Или вообще предполагать, что одно не существует без другого.
Высокоразвитая цивилизация имеет НОВУЮ ФИЗИКУ.
Однако люди не освоившие хотя бы старой физики не в силах вообразить последствия и спектр возможностей.
452 699825
>>699811

>Высокоразвитая цивилизация имеет НОВУЮ ФИЗИКУ.


Это лишнее, без этого тоже очевидно что поиск аналога Земли в радиусе "десятков парсек" по радиоизлучению это бред.
Хватит Юпитера в качестве примера, который на некоторых частотах время от времени фигачит сильнее Солнца. Очевидно что не нашли бы инопланетян по сопоставимому радиоизлучению, только если бы они специально не передавали через радиотелескопы.

Если кто не согласен, то тащите сюда статейки с расчетами. Или сами посчитайте как будет выглядеть на межзвездном расстоянии суммарный сигнал допустим от вещалок FM диапазона.
453 699827
>>699825

>сигнал допустим от вещалок FM диапазона.


Почему именно вещалки? А потому что их много и они везде лезут. Достаточно включить спектроанализатор даже без антенны, чтобы увидеть вздутие в этом диапазоне. Правда вверх они вряд ли фигачат с такой силой, но будем считать что они будут на 180 градусов равномерно. Мощность у них относительно небольшая, ну допустим в среднем 10 кВт и вещалок этих 50 штук на 20 миллионов населения (примерно как в Москве). Сколько народу живет с одной стороны Земли? Ну где-то 1.5 миллиарда более-менее, плюс-минус сколько-то. Это значит что одна сторона земли в этом диапазоне фигачит в этом диапазоне в сумме на 40 МВт, если с огромным запасом.
Теперь оценим как этот сигнал будет выглядеть на Марсе. Как известно, ослабление в открытом пространстве пропорционально квадрату расстояния. До Марса около 50 млн км, поделим 40 МВт на квадрат расстояния и получим 0.02 мкВт. То есть вся Земная мощь на таком расстоянии на участке 88-108 МГц сравнима с передатчиком 0.02 мкВт на расстоянии 1 км. Ну, успехов принять такое без огроменных антенных полей. Можно еще сравнить с уровнем естественных шумов, но мне сейчас лень.

Так что если какие-то "ученые" в своих статейках писали обратное, то это лженаука. И анон, который тут бегает с криками о том что "инопланетян нету", просто шизик. Не важно есть ли они там, без них даже лучше, нам своих мудил хватает. Но очевидно что отсутствие радиоизлучения - это не доказательство.
454 699840
Вы тут спорите а тем временем кто то такую годноту сделал
https://youtu.be/saWNMPL5ygk

Обсуждение иди
Надеюсь анону понравится
455 699849
>>699840
Ерунда. Лучше обсудить что-то более предметное, только похоже что тут никто даже школьную физику не осилил.
456 699850
>>699827
Продолжу.

>0.02 мкВт на расстоянии 1 км.


Теперь сравним это с природным шумом - реликтовым излучением, на фоне которого в идеале можно было бы принять сигнал, если бы рядом с Землей не было Солнца.
Как известно, это излучение соответствует температуре 2.7 К, а суммарная мощность шума в полосе частот kTΔF=-150 дБВт на диапазоне 88-108 МГц. Тут удобнее писать в дБ, а то числа маленькие сильно.
Теперь вернемся к суммарным 40 МВт на межпланетном расстоянии. Ослабление в свободном пространстве на таком расстоянии 226 дБ, а 40 МВт это 76 дБВт.
Если приемная антенна всенаправленная, то примется мощность 76-226+3=-147 дБВт. Уже выше уровня реликтового излучения! А если взять направленную антенну то будет еще лучше. Но если подставить межзвездные расстояния, то на одном световом годе будет уже -259 дБВт в изотропной приемной антенне. Итого коэффициент направленного действия приемной антенны должен быть на уровне 100 дБ. Неплохо так. Можно прикинуть размер такой антенны по формуле для параболической антенны - будет порядка 100 км, с подобной антенной можно было бы уверенно обнаружить такое излучение, оно бы на 3 дБ торчало бы над остальными шумами. Но это только теоретически, при отсутствии других источников излучения кроме реликтового. И еще вопрос как доказать искусственное происхождение.
Выходит что в УКВ диапазоне Земля излучает слабовато. Может в более высокочастотных диапазонах будет получше, но они широкие и сигнал по ним будет размазан равномернее.

Так что с высоты моего дивана видно что на Земле сейчас вряд ли получилось бы обнаружить вторую Землю по радиоизлучению, даже будь она у Альфы Центавра. Обнаружение возможно только если друг другу целенаправленно радиотелескопами сигналить.
457 699853
>>699850
Можно тупенькому мимокрокодилу понять что за дБВт?
458 699855
>>699853
Это всего лишь десять десятичных логарифмов от мощности в ваттах, чтобы фемтоватты не писать.
То есть 0 дБВт=1 Вт, 10 дБВт=10 Вт, 20 дБВт=100 Вт, 30 дБВт=1000 Вт. Там где без приставки Вт - это просто в разах, по мощности. Так часто удобнее и нагляднее, в добавок умножение с делением заменяется сложением и вычитанием.
459 699856
>>699855
А, не видел такого. Я так понимаю -150дБВт это как 10Е-15Вт? Мне через Е больше понятно, честно говоря.
460 699859
>>699856
Да, все так.

>Мне через Е больше понятно, честно говоря.


Наверное дело привычки. Мне тоже раньше это не особо заходило, а потом стал чаще пользоваться измерительными приборами с логарифмическими шкалами и прочувствовал что так удобнее.
461 699866
>>699796
Ты пернул в лужу, а по сути ничем не возразил. Молодец.
462 699888
>>699790
Ты сказал?
463 699889
>>699855

> Так часто удобнее и нагляднее


Нет блять, удобнее и нагляднее в системе СИ, а не очередных шизо-фанфуриках
464 699890
>>699781

> Мне приснился сон, в котором на Землю упал метеорит, что заставило задуматься о том, что было бы, если бы это реально произошло.


Все зависит от размера. Стометровый астероид разъебет город размером с Москву. Километровый устроит локальный апокалипсис в крупном регионе и заметные последствия для всей Земли в виде похолодания на годик. Десятикилометровый даст такого охуенного пенделя всей цивилизации, что мало не покажется: девятибалльные землетрясения по всему земному шару, пожары от падающих осколков, похолодание на 10-20 градусов на несколько лет и темные сумерки на столько же, кислотные дожди, миллиарды трупов, полная пизда.
465 699896
>>699866
Вот так и выглядит обосрамс, анончики.
466 699898
>>699825

>Это лишнее


Если она ВЫСОКОРАЗВИТАЯ - это данность.
Но разумеется саму идею поиска инопланетного радио можно убить и более простыми доводами.
467 699900
>>699896
Это понятно, но отвечать ему не стоило, такие всегда за собой хотят последнее слово.
468 699902
>>699900
Это ж замечательно, пусть еще и изваляется, теряя в процессе остатки иллюзий о своей правоте и безоговорочной победе.
469 699914
>>699888
Логика сказала.
>>699896

>пук

470 699916
>>699849
И что?
Ну не осилил, что теперь про космос нельзя попиздеть.
Мы можем пиздеть, а ты будешь нам считать например.
fieldimagea231.jpg445 Кб, 1634x1080
471 699924
Теоретически, можно ли трахнуть черную дыру членом галактических масштабов?
472 699930
>>699924
У NGC 4889 размеры центральной СЧД в 20 миллионов раз меньше размеров галактики.
473 699949
>>699924
если член чёрный - да
474 699953
>>699924
Всунуть можно, высунуть - нет.
475 699957
>>699924
Черные дыры - не какие-то потаскушки, это высшие особы, нельзя просто сунуть-выйти-и-уйти, если соберешься возлежать - надо будет расписаться. Колечко ей подаришь аккреционненькое и будете жить вместе вовеки веков.
476 699980
>>699949
Так-то он схлопнется и почернеет из-за своей огромной массы, всё прям как ты хочешь. Но при этом сам превратится в дыру, а это уже пропаганда нетрадиционных отношений. Путин накажет!
477 700055
>>699924
Черная дыра — не дыра, это область пространства-времени. Короче, представь себе точку с настолько большой гравитацией, что всё, что находится возле неё на определенном расстоянии, притягивается к ней навсегда. Вот эта "дыра" и есть это пустое пространство.
478 700063
>>700055

>пространства-времени


А это что такое?
479 700065
>>700055

>Короче, представь себе точку с настолько большой гравитацией, что всё, что находится возле неё на определенном расстоянии, притягивается к ней навсегда.


Протон?
480 700068
>>700065
Протон не притягивает навсегда, его гравитация мала.
481 700072
>>700068
Где находится граница между "мала" и "достаточна", и по какой причине именно там?
hjsrfgjsdgj.jpg107 Кб, 1300x1144
482 700078
>>700068
Вообще то притягивается сильным взаимодействием. Прикол в том, что не навсегда пикрелейтед.

Ядерный распад сильное и слабое взаимодействие
Излучение хоккинга для гравитации


Мы чёрные дыры на самом деле не понимаем. Нельзя что то там изучать снаружи, не понимая что внутри.

Может чтобы вытащить что-то из чёрной дыры, нужен "нейтрон", который проходит сквозь чёрную дыру и разламывает её пополам.
483 700081
>>700078

>и разламывает её пополам.


А от этого Вселенная случайно сколлапсировала в точку.
484 700087
Кстати, о черных дырах. Значит ли совпадение изображения тени ЧД с предсказанным, что современные представления о ЧД подтверждены наблюдениями? Или все еще есть место для альтернативных теорий, отрицающих само существование ЧД?
485 700088
>>700072
Там, где образуется радиус Шварцшильда. Из которого уже ничто не может вырваться.
Для массы протона его размеры слишком велики, чтобы схлопнуться в сингулярность.
Но можно напихать сколько-то протонов в кучу, чтобы получить быстроиспаряющуюся ЧД. Вполне конкретое их количество.
Были давно визги, что мол БАК будет черные дыры порождать.
486 700098
>>700088
Протон же нужно еще сжать нехуево.
Чтобы из него струны повылазеля

Надо строить коллайдер на орбите астероидов, поддерюиваемый опорами на астрероидах. А что фермами соед ним астероидный пояс, поставим опоры и проложим трубу. Роботы проложат. Время строительства вашей постройки 1000 лет, милорд.

Ну и сделаем свою черную дыру обьяснив парадокс ферми. Разве не благодать?
487 700099
>>700098
А какая у протона имеется собственно квантовая информация внутрях? Я ноль в квантмех, поэтому скорей всего спрашиваю хуйню, но допустим у нас есть способ сжать один протон до размера, чтобы он превратился в черную дыру, которая конечно очень быстро испариться. Что от такой дыры останется? Проебется ли информация протона?
488 700100
Гравитационный радиус обычных астрофизических объектов ничтожно мал по сравнению с их действительным размером: так, для Земли rg ≈ 0,887 см, для Солнца rg ≈ 2,95 км. Исключение составляют нейтронные звёзды и гипотетические бозонные и кварковые звёзды. Например, для типичной нейтронной звезды радиус Шварцшильда составляет около 1/3 от её собственного радиуса. Это обусловливает важность эффектов общей теории относительности при изучении таких объектов. Гравитационный радиус объекта с массой наблюдаемой вселенной был бы равен примерно 10 миллиардам световых лет[3].

О посоны, то есть 2/3 сжатия материи это просто убирание всего лишнего пространства между протонами?
А 1/3 это уже сдавливание самого протона.
А что говорит квантмех на то, что если бы мы сблищили между собой кварки? Или квантмех такое посчитать не может?
489 700101
>>700098
Зачем трубу? Там вакуум, так пуляй, главное не промахнись.
Опять же, никакой нормальной ДЫРЫ нам не проковырять, она будет испаряться быстрее, чем мы сможем заставить ее аккрецироать. Настолько быстрее, что она просто сразу ебанет после образования, не прожив сколько-нибудь заметного времени.

Именно поэтому мы не видим маленьких черных дыр нигде. Не то что массой с землю, даже и с юпитер, или с красный карлик.
490 700102
>>700087
Да, значит. Но места для альтернативных вариантов и до того мало было. Остались несколько экзотических конструкций, вроде гравастара, которые должны были бы полагаться на квантовые эффекты. Наблюдаемые снаружи различия там появляются лишь совсем рядом с горизонтом (или тем местом где его нет). LIGO по большей части отсёк это всё. Все эти варианты полагаются на гипотетическое нарушение ТО, ещё не обнаруженное.
491 700103
>>700100
Нет, это уже сдавленые до предела протоны надо додавить еще до трети радиуса и вот тогдаааа
492 700108
>>700103
дада проебался не проснквшись еще написал
493 700126
анон, имею скайвотчер 809 но со сломаной азимутальной монтировкой, где купить монтирвоку не по конским ценам? Почему на оф сайте св телескопы с монтировками дешевле голых монтировок?
494 700127
>>700055

> Черная дыра — не дыра, это область пространства-времени. Короче, представь себе точку с настолько большой гравитацией, что всё, что находится возле неё на определенном расстоянии, притягивается к ней навсегда. Вот эта "дыра" и есть это пустое пространство.


Какая же наркомания.
495 700128
496 700129
>>700087

>Кстати, о черных дырах. Значит ли совпадение изображения тени ЧД с предсказанным, что современные представления о ЧД подтверждены наблюдениями?


Да.

>Или все еще есть место для альтернативных теорий, отрицающих само существование ЧД?


Почти нету, чем больше становится данных, тем более маргинальными становятся эти альтернативы.
497 700130
>>700099

>Что от такой дыры останется?


Ничего.

>Проебется ли информация протона?


По идее должна каким-то образом остаться, чтобы не противоречить квантовой механике. Но каким образом - непонятно. Излучение Хокинга тепловое, из него никакой инфы о содержимом черной дыры нельзя вытащить. Видимо, существуют какие-то иные способы сохранения информации, которые пока не открыты.
498 700164
>>699896
Научный диспут, как в средневековом универе, с обвинением в ереси и переходом в драку (веселое было время).
499 700184
>>700164
Нашел где чистоплюйствовать.
500 700197
>>697201
это как: почему не едят говно, чтобы наесться? Потомушто это отходы, бля
501 700202
>>700197
Они ж тепло выделяют, бля. Тепло можно использовать для совершения работы, бля, разве нет, бля?
502 700229
>>700202
а кроме тепла ещё говённые излучения и частицы, теплоноситель облучён будет
503 700278
>>700202
Выход тепла у отходов относительно малый, зато они люто ебашут гамма квантами и выделают много нехорошего газа. Гаммы кванты вызывают активацию материала конструкции и теплоносителя, в итоге конструкции быстро теряют прочность, а теплоноситель портится. Даже в обычной воде не очень хорошо отстаивать, ибо вода разлагается на водород и кислород, которых потом могут в бабах. В добавок в воде(и не только в ней) возможен электронный захват и генерации нейтронов, которые наводят радиоактивность на обычный материал.
504 700306
>>700229
Это в классическом исполнении, да ведь?

>>700278
Так почему бы не приспособиться? Бассеин такой, там все говны бурлят, ты водороды нахаляву собираешь.
Ну я вот дебил, я вижу только профиты.
Все говорят что это пиздец неудобно - я говорю - так в бассеин ебаните это все, оно вам тепло и энергию нахаляву будет давать.
В чем подводные? Ах да, вот в них и есть. Это ж беспроигрышный.
505 700315
Есть инфа от знающего рептилоида.
Путешествовать меж звёзд намного проще, чем кажется на первый взгляд.

Для этого всего лишь нужны разработки в области не нанотехнологии.

Вместо того, чтобы отправлять армады кораблей к другой звезде, туда отправляют маленький зонд.
Который уже на месте, из местных материалов печатает эдакие 3д-принтеры, вся эта масса растёт в геометрической прогрессии, питаясь энергией звёзды.
На месте уже производится вся инфраструктура, тела для разумных существ.
Далее просто пересылается сознание со световой скоростью.

Подводные?
506 700316
>>700315
Таким Макаром можно всю галактику засрать запросто
507 700332
>>700315
И хули там делать всей этой инфраструктуре и полчищам клонов??
508 700337
>>700306

>Это в классическом исполнении


а в каком ещё можно? в нетрадиционном чтоли?
509 700338
>>700278

>Гаммы кванты


не знаю чё такое зарыто в европе, но это зарытое говно ебашит на весь мир ГОМО квантами
510 700339
>>700306
кстати вот что ещё вспомнилось..

Мой комбинат ебашит вообще адовые изотопы.
Ну такой вот примерно рецепт усредненный, потому что вариаций масса.
Берется отработанное ядерное топливо, оно не остывает в бассейне передержки, остужать - это не про мой комбинат. Он берет это ядерное топливо, вываливает его в бочки и начинает сушить. Добавляет в него огромное количество кислоты, цемента, песка для вязкости, рутений сверху. Все это сушится до дыма. Потом бросается в местную речку. Потом служба дозиметрического контроля щедро пиздИт местным жителям что фон в пределах нормы. При этом пиздЯт со счетчиком гейгера в руках. ПиздЯт и приговаривает полушепотом "ух бля". При этом у них на лбу аж пот выступает. Любезно мне иногда предлагают захоронить отходы, но я отказываюсь. Надо ли говорить о том какой дичайший фон потом? Фонища такая, что обои от стен отклеиваются.
511 700358
>>697202
Почему тогда фотон света не может вырваться из горизонта событий?
512 700359
Помимо Урана и Плутония есть же другие изотопы ядерные. Почему их не хуячат в космосе?
Какой-нибудь Калифорниевый реактор был бы очень мощный же врлде?
Это из-за того чтобы надо чтобы оно долго работало а не мощно?
513 700367
>>700306

>ак почему бы не приспособиться? Бассеин такой, там все говны бурлят


Так не бурлят же!
Славик, ты думаешь, там дяди тупее тебя не умеют энерговыделение считать? Или глазок нет у них?
Чтобы с высоким КПД получать энергию, нужна большая бурлящая способность. Иначе это все - только согреть воду в луже без профита.

Бурлят только свежие, полные топлива невыгоревшего ТВЭЛы, в реакторе. И вот там все как ты хотел - турбина блять, пар нахуй, и дешевое электричество в ризетках.
514 700372
>>700359
А ты никогда не задумывался, как именно выглядит нахуячивание изотопов?

В деталях, тассть? Нет, навалить дерьма в реактор - этого недостаточно.
Потом, нужно достать облученное нечто. Это нечто растворить в кислотах, поеботах, чтобы выделить химический элемент, очистить и сконцентрировать его из едкой и ядовитой, разъедающей все вокруг горячей чачи. А потом из выделенного элемента выделить нужный изотоп. Которого там децл на тонну. Сконцентрировать хотяб до нескольких процентов - то есть в куче ненужного говна будет несколько процентов изотопа.
И затем запаковать этот концентрат в капсулы, этими капсулами набить реактор, и цыкл повторяется.

Даже на земле это делать геморрой. В космосах роботами еще геморнее. Не говоря про запуск туда портящихся от радиации и кислот мясовеков.
image.png2 Кб, 576x39
515 700374
>>700358
Потому что всё, что идёт дальше плутония - это элементы, полученные условно в ускорителях частиц. Чем дальше порядковый номер ==> тем тяжелее его получить ==> тем дороже он будет и период полураспада, к слову, тоже меньше
516 700375
>>700372
Это сложно, но не невозможно? Мы просто мало ядерными вещами занимаемся, верно?
517 700376
>>700367
А просто греть воду без кипячения и отапливать дома, например? Тоже же тепло. Халявная энергия.
image.png513 Кб, 720x576
518 700378
>>700358
ты путаешь причину и следствие, это горизонт существует из-за того что фотон не может вырваться до определённого радиуса
519 700381
>>700376
водогрейный ядерный котёл на радиоактивных отходах, ... да ты охуел, Исумбоси?! почитай про ВВЭР, там и то пар
520 700382
>>700378
Это вроде прям определение?
521 700384
>>700381

>Исумбоси


Пиздец, я лет тридцать не слышал этого слова.
Спасибо.
522 700387
>>700376
Еще раз - у отработки тепловыделение падает очень быстро. Она просто не нагреет воду до вменяемой температуры.
Именно поэтому это дерьмо вынули из реактора и оставили отлеживаться. Не греет.

Если у тебя есть много отработанного ядерного топлива, то у тебя есть и АЭС рядом, голубчик!
Для справки, АЭС только одну треть выделяемой топливом теплоты превращает в электроэенергию.
Две трети - рассеиваются в воздух. Там теплоты СТОЛЬКО, что хватит на мегаполис. Но строить мегаполис вокруг реактора никто не спешит.
image.png228 Кб, 1048x770
524 700403
>>700395
ну, калифорний очень хитрый метал, из-за радиоктивности его цена оооочень сильно скачет
image.png4 Кб, 501x54
525 700406
>>700403
Странно... "самый дорогой металл"? А как же другие элементы?
526 700407
>>700406
они только в теории существуют, как и сфера дисона
527 700410
>>700407

>в теории


Такое...

Конечно, навряд ли получится получить эти металлы в достаточном количестве, однако атомы этих элементов получить возномжно.
528 700414
>>700315
Маняфантазер, спок.
image.png9 Кб, 611x152
529 700415
>>700410
ля, анончик! я тут новый химический элемент изобрёл, пока срал в сортире, проктически получать не стал, ибо он пиздецкий радиоктивен, а получить один атом невозможно. Он получается сразу по полкилограмма, такшто пруфов кроме формулы не будет. Стоимость его невелика в январе, но к декабрю пиздец как вырастает. Применение очень ограничено - нужен только если желаешь облучить за 5 минут всю плонету размером с юпитер, на это хватит 200 грамм металла.
530 700416
>>700358
Потому что вторая космическая скорость за горизонтом событий равняется скорости, большей скорости света.
531 700419
>>700416
А с чем это связано?
532 700420
>>700419
ахахаха! с тем что скрость света постоянна!
533 700467
>>700419
С гравитацией.
534 700486
>>700419
а более лутшее почитай про радиус Шварцшильда, и на лурке очень легко изложено
535 700495
>>700375
Это дорого настолько, что лучше даже не начинать.
В космосе неподалеку от стабильной желтой звезды и так дохуя энергии, только панели разверни, которые и дешевле, и проще, и выхлопа с них сразу и дохуя.
image.png115 Кб, 1200x864
536 700517
>>700387
Тогда еще два вопроса:
почему отработку не перефаршмачивают чтобы минимизировать долго хранимые говеные изотопы, а просто все в бочки и хранить?
и почему ёбаные фрицы против ядерки, но типа за зеленую энергию? АЭС это разве не самый эффективный способ уменьшить выхлоп парниковых газов и при этом иметь дофига электричества?
537 700518
>>700415
А вот мы шутим, а невозможно ли иметь йоба-элемент, пусть и энергетически затратно получаемый, который бы очень легко но контроллируемо распадался и давал дофига энергии?
Типа Уран+, чтобы в космосы гонять удобно было.
Я плохо разбираюсь, просто хочется интересностей.
;kknkn ,.jpg109 Кб, 500x476
538 700545
>>700518 есть конешно! это Двачевий, но профита от него мало
а вот тебе интересность, йоба-объекты типа RSGC2-18, работает на Н и Не, но их надо ОЧЕНЬ ДОХУЯ, чтоб они заработали, настолько дохуя что капс лока микроскопически недостаточно. Я далеко не спец, но думается мне што ядерные топливы используются как обычные газ и нефть. То есть взкипячивают водичку, а та всё делает по старой схеме, либо помаленьку юзается в приборах - детекторы и приёмники. Нет такого "ебатьэлемента" чтоб и на хуй сесть и рыбку съесть. В кинах и играх только. Прочитай https://mipt.ru/dcam/students/books/fiction/otvet.php \\\\мне в 12 лет очень пропёрло, я спиздил книгу в магазе, кстате
539 700572
>>700517

>почему отработку не перефаршмачивают чтобы минимизировать долго хранимые говеные изотопы, а просто все в бочки и хранить?


Дык перефоршмачивают же!
Но есть нюанс. Чем меньше светится отработка, тем дешевле перефоршмачивать.
А чтоб она меньше светилась, ее надо выдерживать в хранилищах по 60 лет.
Не напомнишь, когда у нас вообще появились первые АЭС?
Фрицы просто ебанавты, хуй им в сраку. Это природа им мстит за Гитлера, лишая мозгов. Ну и еще - у фрицев самые развитые и населенные, промышленные регионы находятся на границе Франции. У которой атомок дохуя и больше.
540 700613
>>700495
На орбите Сатурна широченные солярки размером с бассеин дадут тебе пару киловатт всего. Столько массы тягать туда уже не комильфо. Джуно уже был чуть ли не больше солярками чем зондом, а это Юпитер.
Надо прям вообще рядом со сраной звездой мутиться чтобы лепездричество выкачивать, чуть дальше зачешешь - уже соснёшь.
541 700663
>>700613
ох уж эти солнечные понели вокруг солнца, бля, а электричество как передаётся от них? по космическому воздуху? проводов не надо?
542 700666
>>700663
Микроволнами или внезапно сделать "провод" из плазмы из тяжелых ионов.
543 700687
>>700613
Так давай Плутон освоим, делов-то.
Ничего, что любая тепловая хуета требует конских РАДИАТОРОВ, и тебе понадобится или хайтечную ебу тащить с риском фейла, или городить такие же огромные рассеиватели теплоты.

Там вакуум, можно к Сатурну сколько угодно солярок притащить на тросу, произведя их в более энергетически жырных местах.
Зато без рисков радиоактивных аварий, всеобщей обесточки - солярки все разом не пизданутся, в отличие от.
Ну а в темную мрачную стылую жопу вне окресностей звезды придется тащить ссобой радиоактивную бяку, да не одну - резервирование, надежность. Поэтому окрестности Плутона - такая себе космическая недвижимость.
16340908358920.jpg171 Кб, 700x943
544 700690
Вспомнил трилогию "Задачи трёх тел" и задумался.
Вот, например, сейчас у меня состоится контакт с настоящей инопланетной цивилизацией. Причём не людьми с другой планеты, а вообще другими существами. Допустим даже, что они разработали некий коммуникатор, способный передать свою информацию мне в компьютер прямо на аккаунт в ТГ в виде текстового сообщения на русском языке, неотличимом от переписки с обычным человеком. Какие у нас могут недопонимания на уровне общения? Если подумать, начнут они общаться и каждый из нас будет воспринимать другого как более-менее одинакового габарита существа, а на самом деле, может они не больше мыши каждый или, наоборот, один стандартый среднего роста человек для них всего-лишь какой-нибудь кот ростом - ведь мы тогда даже о размерах и расстояниях не сможем поговорить
545 700709
Почему иногда небо розово-фиолетовое, особенно в зиму и осень?
Почему вечера летом оранжевые?
546 700714
>>700690

>ведь мы тогда даже о размерах и расстояниях не сможем поговорить


Именно для этого Карл Саган пририсовал голую бабу в масштабе к табличке на Вояджере
547 700720
>>700714
чтоб инопланетяне начали безудержно фапать
548 700751
У меня наверное максимально тупой вопрос.
Если свет это и волна, и частицы, почему два скрещенных фонаря продолжают светить дальше? Частицы же начнут сталкиваться друг с другом и их поток станет слабее, а волны погасят друг друга и начнут идти в другом направлении.
549 700756
>>700751
один фонарь хуярит частицами, а другой волнами, вот и все дела
Безымянный.png19 Кб, 1331x168
550 700805
Как плотность и температура могут быть бесконечными? И с чего вообще взяли, что они были бесконечными?
551 700811
>>700805
По этой теории все вещество, которое сейчас есть во вселенной, было сжато в сферическую математическую точку в вакууме. Проверенные аксиомы говорят что в таком случае плотность и температура будут настолько высокими, что проще называть их бесконечными, чем попытаться высчитывать все это.
Не нравиться эта теория, можешь предлагать свою. Никто же не против. Ну или попытайся вычислить плотность и температуру сам. Но повторяю речь о математической точке, радиус которой приблизительно равен нулю.
552 700813
>>700811
*нравится
553 700847
>>700811

>проще называть их бесконечными


Бесконечность и огромное число это совершенно разные понятия

>радиус которой приблизительно равен нулю.


А это как узнали?
554 700851
>>700709
Солнечный свет рассеивается так в атмосфере.
555 700853
>>700751

>Если свет это и волна, и частицы


Это поток квантовых частиц, фотонов, и все их волновые свойства обусловлены их квантовой природой.

>Частицы же начнут сталкиваться друг с другом и их поток станет слабее


Не станет, потому что поток очень сильный, в нем очень много фотонов.
556 700854
>>700805

>Как плотность и температура могут быть бесконечными?


Никак, в природе не бывает бесконечностей. Когда у тебя в уравнениях вылезают бесконечные величины, это значит, что они уже не работают - по крайней мере, в тех условиях, в которых они дают бесконечности. Значит, теория в этих условиях ломается и не может адекватно описывать явление. Значит, нужна другая теория. В случае сингулярности, в которой на самом деле конечно же никаких бесконечностей нет, а есть планковская плотность, ломаются все существующие физические теории, включая общую теорию относительности. Для описания сингулярности нужна квантовая теория гравитации, а ее еще не построили и никто не знает, когда ее построят.
557 700864
>>700854
а вот если бы у сингуляторности была отрицательная температура, что-то типа абсолютного нуля, а может быть и ниже, то она (сингулярность) была бы раза в два бесконечно плотнее. Какой замечательный парадокс, я даже закурил...
558 700868
>>700811
ну даже число грэма в степени гугол, хоть и охуеть как велико - всё равно жалобно посасывет у бесконечности. И это пиздец как грубо большое число называть бесконечностью. Там все базоны и кварки просто распидорасит, от такой температуры. Даже струнам придёт пизда.
559 700882
>>700864
Нет там никакой бесконечной плотности.
560 700900
>>700882
конешно нет, это даж звучит както антинаучно и дико.
561 700938
562 704817
что случится с Новыми Горизонтами после того, как он выполнит свою миссию? Его можно будет направить обратно?
563 705484
>>704817
Полетит себе дальше.
На то, что бы раззвернуть его в обратную сторону, нет ни массивных обьектов, ни топлива.
Тред утонул или удален.
Это копия, сохраненная 5 ноября 2022 года.

Скачать тред: только с превью, с превью и прикрепленными файлами.
Второй вариант может долго скачиваться. Файлы будут только в живых или недавно утонувших тредах. Подробнее

Если вам полезен архив М.Двача, пожертвуйте на оплату сервера.
« /spc/В начало тредаВеб-версияНастройки
/a//b//mu//s//vg/Все доски